74
PART 1: NEGLIGENCE Negligence: failure to heed a duty of reasonable care owed to another that causes injury to that other Prima Facie Case 1) P has suffered an injury 2) A owed a [reasonable] duty to a class of persons including P to take care not to cause an injury of the kind suffered by P 3) A breached that duty of care 4) A’s breach was an actual and proximate cause of P’s injury INJURY 1) Physical harms (bodily harms, damage/destruction to tangible property) 2) Economic loss/loss of wealth – more complex 3) Emotional distress – more complex DUTY 1) DEFINITION: a) Degree of care which an ordinarily reasonable and prudent person exercises under the same/similar circumstances. Should exercise care necessary to avoid injury to others reasonable person standard. b) Requires negligence P to est that D owed P, or a class of persons including P, an obligation to take care not to cause injury that P suffered 2) Objective test. 3) Duty owed to foreseeable P. 4) Duty is a ques of law for court – must balance 3 factors p.79: a) Relationship between the parties b) The reasonable foreseeability of harm to the person injured c) Public policy concerns 5) Easy duty cases: D and P satisfied that D owed P duty – won’t spend much time on the duty issue 6) No duty to economic losses unconnected to physical harm; exception – malpractice, cause someone to lose $ 7) EVOLUTION OF DUTY RULES a) Winterbottom v. Wright (1842): established privity rule; carriage builder sued by employee of company that bought carriage. Ct rejected Winterbottom’s claim, based judgment on remoteness of event from careless act Wright committed. b) Thomas v . Winchester (1852): Winterbottom rule in the US limited, imminently dangerous products. Thomas sued Winchester for mislabeling bottle of poison, causing her to be accidently poisoned. Ct ruled for P, danger could be foreseen. c) Heaven v. Pender (1883): no more privity reasonable foreseeability. i) Reasonable foreseeability: so long as physical harm to P is a reasonable foreseeable consequence of an actor’s careless conduct, the actor owes duty to take care not to cause such harm to P (1) If foreseeable that someone could get injured by what you’re doing, you have a duty to take care in what you are doing (2) Would a person of ordinary sense recognize that, if she pursued the conduct at issue w/o vigilance for physical well-being of others, her Amanda Lee, Hershovitz – Torts, Fall 2009 1

Torts outline

Embed Size (px)

Citation preview

Page 1: Torts outline

PART 1: NEGLIGENCENegligence: failure to heed a duty of reasonable care owed to another that causes injury to that other

Prima Facie Case1) P has suffered an injury2) A owed a [reasonable] duty to a class of persons including P to take care not to cause an injury of the kind suffered by P3) A breached that duty of care4) A’s breach was an actual and proximate cause of P’s injury

INJURY1) Physical harms (bodily harms, damage/destruction to tangible property)2) Economic loss/loss of wealth – more complex3) Emotional distress – more complex

DUTY 1) DEFINITION:

a) Degree of care which an ordinarily reasonable and prudent person exercises under the same/similar circumstances. Should exercise care necessary to avoid injury to others reasonable person standard.

b) Requires negligence P to est that D owed P, or a class of persons including P, an obligation to take care not to cause injury that P suffered

2) Objective test.3) Duty owed to foreseeable P.4) Duty is a ques of law for court – must balance 3 factors p.79:

a) Relationship between the partiesb) The reasonable foreseeability of harm to the person injuredc) Public policy concerns

5) Easy duty cases: D and P satisfied that D owed P duty – won’t spend much time on the duty issue6) No duty to economic losses unconnected to physical harm; exception – malpractice, cause someone to lose $

7) EVOLUTION OF DUTY RULESa) Winterbottom v. Wright (1842): established privity rule; carriage builder sued by employee of company that bought

carriage. Ct rejected Winterbottom’s claim, based judgment on remoteness of event from careless act Wright committed.

b) Thomas v . Winchester (1852): Winterbottom rule in the US limited, imminently dangerous products. Thomas sued Winchester for mislabeling bottle of poison, causing her to be accidently poisoned. Ct ruled for P, danger could be foreseen.

c) Heaven v. Pender (1883): no more privity reasonable foreseeability.i) Reasonable foreseeability: so long as physical harm to P is a reasonable foreseeable consequence of an

actor’s careless conduct, the actor owes duty to take care not to cause such harm to P(1) If foreseeable that someone could get injured by what you’re doing, you have a duty to take care in

what you are doing(2) Would a person of ordinary sense recognize that, if she pursued the conduct at issue w/o vigilance

for physical well-being of others, her conduct would pose a meaningful risk of physical injury to persons in the position of P?

d) MacPherson v. Buick Motor (1916): struggle with privity rule over, sets privity aside – obligation to one another cannot be limited by agreement. Ruling is not no privity. Tort will be solved by tort. Case is about relationship between contract and tort law.i) Principle of Thomas not limited to imminently dangerous products. Cardozo’s 3 part test:

(1) Reasonable certainty of danger(2) Know that used by others(3) No further inspection likely

ii) Cardozo’s 3 part test is now reasonable foreseeability iii) Cardozo’s reasoning in case:

(1) If nature of a thing is such that is is reasonably certain to place life and limb in perid when negligently made, it is then a thing of danger.

(2) If to danger there is added knowledge that thing will be used by person others than purchasers, and used without new tests, then irrespective of contracts, manufacturer is under duty to make thing carefully.

iv) Rule: One who invites another to make use of an appliance is bound to exercise of reasonable care.v) If danger was to be expected as reasonably certain, there was a duty of vigilance.

Amanda Lee, Hershovitz – Torts, Fall 2009 1

Page 2: Torts outline

e) Reasonable person standard : Whether injury of the type suffered by P was foreseeable to a reasonable person in the position of D.i) Mussivand v. David : D could forsee that being infected at the time he had sex with P’s wife, would infect P. D’s duty

ended when P’s wife knew she was infected.(1) D has duty to use reasonable care to avoid infecting others.(2) Public policy: health of people is an economic assets. Laws of state framed to protect people.(3) Additional policy issue: Cts don’t want adultery.(4) Amatory torts – “anti-heart balm” statutes

(a) No negligence for:(i) Criminal conversation (adultery)(ii) Alienation of affections(iii) Seduction

ii) Doe v. Moe: Moe broke D’s penis. Doe sued that Moe had duty of care while having sex with him. (1) Policy: Cts don’t want more cases like this.

8) QUALFIED DUTIES OF CAREa) Affirmative duty/duty-to-rescue and protect

i) Unreasonably failed to provide assistance or protection to Pb) Premises liability

i) Permitted or maintained unreasonably dangerous conditions on property in his or her possessionc) Pure economic loss

i) Acted without reasonable care for P’s economic prospectsd) For these types of cases, duty is “in play”

i) AFFIRMATIVE DUTY/DUTY-TO-RESCUE AND PROTECT(1) Nonfeasance: where one fails to perform at all, doing nothing

(a) P must establish special circumstance to prevail on claim of negligent nonfeasance presence of absence or preexisting relationship b/t P and D.

(2) Misfeasance: one performs duty inadequately/poorly(a) duty to take care not to cause foreseeable injury applies only to misfeasance

(3) No duty to rescue (nonfeasance)(a) General rule : an individual does not have a duty to aid or protect another person, even if he

knows that person needs assistance. But if undertake rescue, must act reasonably.(i) Ex: Osterlind v. Hill p.76: D rented canoe to drunk P. Canoe overturned and D did not help P. P

drowned. a. Ct ruled that D had no legal duty to help P. There was no relationship between D and P.

(b) Exceptions to rule p.79: (i) Exceptions under § 314A of Restatement: special relations giving rise to duty to (a) aid after

knowing one is ill/injured and to care for them until they can be cared for by others; and (b) protect one against unreasonable risk of physical harm p.79

a. Common carrier under duty to its passengersb. Innkeeper under duty to its guestsc. Possessor of land who holds it open to the public is under duty to those who enter in

response to invitationd. One who is required/voluntarily takes custody of another under circumstances to deprive

other of his normal opportunities for protection is under duty.1. Exception applies only while relationship exists2. Duty to take affirmative steps to rescue person in peril when one proceeded against is master

or an invitor or when the injury resulted from use of an instrumentality under the control of Da. From Ayres case – boy injured by escalator in department store and employee’s failure to

stop escalator p.803. Duty to one in peril on D’s premises even though peril created w/o negligence on part of D4. Business is not required to take any action beyond that which is reasonable under the

circumstances; duty to exercise reasonable care under the circumstances(ii) Ex: Baker v. Fenneman & Brown (Taco Bell) p.77: P bought drink at Taco Bell and then fell

backward, hitting head on floor and knocked unconscious and began having convulsions. Then woke up, stood up and fell again, knocked unconscious. P claims no one helped him.1. Taco Bell falls under § 314A – innkeeper under duty to is guests

Amanda Lee, Hershovitz – Torts, Fall 2009 2

Page 3: Torts outline

2. Ct ruled that D had duty to give aid to one who is ill/injured, even though injury due to natural causes, pure accident, acts of third person, or to negligence of P himself.

3. Public policy suggests that D had duty to reasonable care.4. Imposing on businesses a duty to provide reasonable care even when the business is not

responsible will rarely force a business to act in circumstances in which it should not have al ready acted.a. A business won’t just look at a situation and determine whether injury was caused by a

business before helping.(iii) Ex: Farwell v. Keaton p.87: wingman relationship – some courts have held an established friendship

b/t D and P will suffice to support a duty to rescue1. P and D were friends, got beat up. D left P at front door of house. Next morning found dead. D

had duty.(iv) Exception if imminent peril to P caused by D p. 84

1. When actor knows or should know that he has by his own conduct caused the victim to be physically injured and at risk of further injury, or to be in imminent danger of physical harm. Actor has duty to make reasonable efforts to prevent victim for suffering further harm, or to prevent risk of harm from being realized.

(v) Exception if voluntary undertakings: if D has volunteered to protect another from physical injury, property damage, or rescue another from physical peril1. Once rescue is voluntarily undertaken, rescuer has a duty to victim to perform rescue with

reasonable care2. Good Samaritan statutes: shields people who attempt to rescue from liability for negligence

a. Tend to be limited to “off-duty” professionals, like doctors, firefighters, etc.(b) Why no duty to rescue? It’s morally wrong not tot help, but can’t legally enforce

(i) Epstein p.831. Law would violate basic principles of liberalism if it required more of individuals than simply

refraining from injuring others;2. Impossible to draw a principled line b/t duty to rescue someone in imminent peril and a more

general duty of beneficence(ii) Good default rule – rescues are risky

ii) PREMISES LIABILITY: duty you owe depends on the person’s status in relation to your property, in regards to conditions of property, not activities on property.

Who Which Conditions What

Trespasser No permission Attractive nuisance Constant trespassers

No wanton/willful harm Make safe for children Constant trespassers – treat

like licenseeLicensee Permission

Ex: social guests Conditions you know about Or have reason to know,

not same as “should have known”

No duty to inspect

No wanton/willful harm Make it safe or warn of

hidden dangers

Invitee Permission Mutual benefit

Know about Or could discover through

inspection

Make safe or warn of hidden dangers*

*If know likely to get hurt, do more than warn

(1) Classifying status of injured person as invitee, licensee, or trespasser(a) Deter of which status can be jury question, but where the facts are not in dispute the classification

becomes question of law for trial judge(b) INVITEE: person who goes upon premises of another in answer to express or implied invitation of

owner of occupant for their mutual advantage (ex: customer, churchgoers)(i) Duty Owed: owed the highest duty of care – affirmative duty of reasonable care. Landowner must

make reasonably safe or warn of hidden dangers; inspect property of hidden dangers (duty for known dangers, but also duty to find and take care of unknown dangers).

(ii) Ex: Posecai v. Wal-Mart Stores, Inc . : P assaulted and mugged in the parking lot of D’s store. No security present in parking lot. Expert testimony – there had been a few crimes in parking lot but store itself was not a high crime area, even though neighborhood around it was.

Amanda Lee, Hershovitz – Torts, Fall 2009 3

Page 4: Torts outline

1. Ct held that Sam’s club did not have duty to Posecai. 2. Rule: Ct held that businesses do have duty to implement reasonable measures to protect their

patrons from criminal acts when those acts are foreseeable, as determined by a balancing test weighing the foreseeability of harm against the burden of imposing a duty to protect against criminal acts of 3rd parties.

3. 4 approaches of foreseeability:a. Specific Harm Rule – landowner does not owe a duty to protect patrons from violent acts of

third parties unless he is aware of specific, imminent harm about to befall themi. Rule is too restrictive in limiting duty to business owners

b. Prior similar incidents test : foreseeability established by evidence of previous crimes on or near the premisesi. Lead to arbitrary results because applied with different standards regarding number

of previous crimes and degree of similarity required to give rise to a dutyc. Totality of circumstances test : Take additional factors into account – number, nature,

location of prior similar incidentsi. Tend to place greater duty on business owners to foresee risk; P friendly testii. Been criticized as too broad a standard

d. Balancing test : balancing foreseeability of harm against burden of imposing a duty to protect against the criminal acts of third personsi. High degree of foreseeability, greater burden on D, and vice versaii. Ct asked how likely it would be that harm would happen against how hard it was to

prevent – decide whether it’s reasonable to impose burden on store to prevent it.iii. Hershovitz: conceptual error in balancing test – the court is collapsing duty and breach

into one analysis. Decides whether there’s duty by asking whether D’s action was reasonable – that’s breach analysis. Asking if they can take care at low cost isn’t duty analysis – there either is or isn’t a duty, and cost of preventing should be used to determine whether they breached that duty by acting unreasonably (not preventing the harm when it was reasonable to do so). They collapsed it because they could only resolve case as matter of law.

(c) LICENSEE: one who enters upon property of another for his own convenience, pleasure or benefit pursuant to the license or implied permission of owner; permission to be on the land, but not for the landowner’s financial gain – social guests, people invited in, door-to-door charity workers.(i) Duty owed: Duty to inform of or fix known, hidden dangers (don’t have to warn of obvious dangers,

but have to warn or fix if there’s something you know about that the licensee can’t be reasonably expected to know about). No duty to discover dangers you don’t know about.

(ii) The landowner must effectively put the licensee in the same position as the landowner—aware of the known dangers.

(iii) Difference b/t invitation and permission:1. Invitation: conduct which justifies others in believing that the possessor desires them to enter

the land2. Permission: conduct justifying others in believing that possessor is willing that they should

enter(d) TRESPASSER: one who enters upon another’s premises without license, invitation or other right;

enters merely for own purposes, pleasures, or convenience, or out of curiosity, without any enticement, allurement, inducement, or express or implied assurance of safety from owner(i) Duty owed: no duty of care, but must refrain from willful/intentional harm – no duty to trespasser

with regards to conditions of the land, but you can’t set traps(ii) Exceptions:

1. Landowners must take reasonable care to avoid causing injuries to child trespassers who cannot appreciate the dangerous conditions of the property.

2. Attractive nuisance – especially applies for kids (swimming pools) but could also apply if there are healing springs on your property that draw the elderly – have duty to warn of danger if they turn out to be way hotter than they are, or they look otherwise ok but are poisonous.

3. Landowners have a duty to constant/persistent trespassers when the landowner knows, or has good reason to know, of the presence of trespassers on his property. a. Otherwise, usually not liable to take care to protect against harms by “natural” conditions,

like trees.b. No-duty rule doesn’t apply when danger is posed by “artificial” conditions created/known

by possessor.(iii) Don’t have to know you’re trespassing to be a trespasser.

Amanda Lee, Hershovitz – Torts, Fall 2009 4

Page 5: Torts outline

(iv) Ex: Leffler v. Sharp: P fell through roof of premises adjacent to inn, by stepping through window.1. P was invitee in inn - both P and D mutually benefited: P got to continue his night of fun and D

got a profit2. P not invitee on roof - invitee who goes beyond bounds of his invitation loses the status and

rightsa. No mutual benefit for P to have D walk out onto roof.

3. P not licensee on roof - D never engaged in conduct signifying to patrons roof area was open – P lacked permission

4. Ct found P was trespasser:a. P entered roof without invitation or permissionb. P not enticed, etc by D

5. Landowner must keep premises reasonable safe – D did. P’s accident didn’t occur within walls of inn. D locked glass door and marked it – done to keep patrons safe.

(2) Person need not own property to be a possessor for purposes of premises liability. p.94 (a) One who is entitled to occupy land can be held liable as a possessor even if he or she does not exercise

that entitlement.(3) If make reasonable warning, off hook for premises liability.(4) ½ the states have abolished the licensee-invitee distinction, creating a duty of reasonable care to all those

who enter property by permission, while retaining the traditional common law rules for trespassers. p.96

9) OBLIGATIONS TO THIRD PARTIESa) Common law rule: one person owes no duty to control the conduct of another, nor to warn those endangered by

conducti) Exception: Duty to control a third person may arise when there’s a special relationship between P and D,

the person whose conduct needs to be controlled, or a relationship between D and a foreseeable victim of that conduct(1) Ex: employer-employee, doctor-patient, custodial relationship: parent-child, warden-prisoner(2) Ex: Tarasoff v. Regents of University of California : P was killed by a man seeing a psychiatrist employed by

D. It is alleged that the patient confided in his psychiatrist his intent to kill P. D reported to authorities but didn’t warn P – duty to have done so?(a) Holding: Doctor doesn’t need to be actually aware that patient is contemplating an attack on someone,

so long as doctor reasonably should have known of the risk.(b) Rule: Once a therapist determines that a patient poses a serious danger of violence to others, he

bears a duty to exercise reasonable care to protect the foreseeable victim of the danger. This duty emanates from the doctor-patient relationship.(i) Patient confidentiality v. safety of possible victim(s)

1. Safety wins out – majority holds that public policy demands a duty as long as risk is reasonably known and foreseeable.

2. Dissent thinks public policy demands the other way – this rule will prevent people from seeking therapy, therapy will be less effective when people do get it, and there will be less crime if we can have more successful therapy, so psychiatrists shouldn’t have duty here – we should really encourage doctor-patient confidentiality in the interests of public safety.

(3) If doctors are aware of a patient having a contagious disease, they must report it to public health officials.(4) Doctors under duty to third parties to warn patients with disorders, like epilepsy, that they may suddenly

become incapacitated while driving, risking injuries to others.

b) Social Host liability: although rule is different in some states, social hosts generally have no duty for injuries caused by people who drank at their homes. i) Common law recognizes that drinker maintains ultimate power and thus obligation to control his own behavior

– best able to avoid foreseeable risks of own behavior.ii) Hosts generally liable if drunk driving guest is a driving-aged minor.iii) Dram shop laws:

(1) Limited to commercial sellers of alcohol(a) They have training in telling how drunk someone is(b) They have a financial incentive to serve as much alcohol as possible

(2) Enacted during late 1800s/early 1900s, leading up to prohibition(3) Enabled dependents to recover compensation for loss of econ support /c their breadwinner had gotten

injured/killed while drunk (4) Modern version focuses on third party victims

iv) Ex: Graff v. Beard: Beard injured on motorcycle after leaving Graff’s party drunk.

Amanda Lee, Hershovitz – Torts, Fall 2009 5

Page 6: Torts outline

(1) Factors to decide whether to impose duty:(a) Foreseeability and likelihood of injury weighed against social utility of actor’s conduct(b) Magnitude of burden of guarding against injury(c) Consequences of placing burden on D(d) Whether one party has superior knowledge of risk(e) Whether right to control actors hose conduct precipitated harm exists

(2) Ct ruled that Graff did not have duty. Beard has ultimate control of own behavior – to decide to drink or not to drink, to decide to drive or not to drive.

(3) Common law focus should remain on drinker as person primarily responsible for his own behavior and best able to avoid the foreseeable risks of the behavior.

10) Calabresi and the Cheapest Cost Avoider p.133: in torts, economic issue is whether dollars spent on preventing accidents are spent efficiently, achieving efficient mix of precaution-taking and injury.a) Calabresi: tort law should aspire to assign liability to cheapest cost avoider – person that can identify and adopt the

most efficient precaution more readily than anyone else.b) Hershovitz: duty to rescue and protect doesn’t quite fit into economic efficiency argument

i) Leffler v. Sharp : Sharp had more access to information about safety of rooftop than Leffler, would’ve been cheapest cost avoider, but ruled for Sharp since Leffler was trespasser.

11) Prosser: duty is a policy question. Do we want to have a duty?

BREACH1) THE MEANING OF NEGLIGENCE

a) Breach of Duty: when D’s conduct imposes unreasonable risk upon another, which results in injury to that otherb) Breach is a question of fact for the jury

i) 4 features of the standard breach instruction – 4 universal features across US jurisdictions(1) Jury instructed that “negligence” (breach) means a failure to use “ordinary care.”(2) “Ordinary care” is defined by reference to a “reasonably careful person” or a “reasonably prudent person,”

and occasionally by reference to a “reasonable person”(3) Instruction directs that jury is to consider whether the D was negliegent in doing something OR in failing to

do something – the jury is to think both about what a D has done and about what the D has not done(4) The jury is required to consider the circumstances in which D acted, and whether D acted with ordinary

care given the circumstancesc) Must show:

i) Proof of what actually happened, andii) D acted unreasonably under the circumstances

Everyone is subject to negligence, unless there’s an exception Some places treat reckless and gross negligence as the same standard. When they’re treated differently, “failed to

take as easy precaution” is considered. Current day courts treat gross negligence and negligence the same. Negligence is easier to prove than recklessness, which is a more demanding standard.

Amanda Lee, Hershovitz – Torts, Fall 2009

Strict LiabilityPingaro v. Rossi

RecklessBeausoleil –

Amtrack case

Negligence Martin v. EvansAdam v. Bullock

Highest degree of care

-Common carriers-Jones v. PAT

Gross Negligence

6

Page 7: Torts outline

d) Negligence: “ordinary care”/”reasonable person” test; must take reasonable precautionsi) Martin v. Evans : D driving tractor-trailer, pulled into parallel parking stop. Got out, came back and no one was

there. But when backing up, felt nudge – P on the ground – had been pinned b/t 2 cars and fell to ground when one car reversed.(1) Issue of whether Evan was credible relied on jury’s determination of ordinary care. Trial ct ended up

issuing for new trial.ii) Adams v. Bullock : P swinging wire while crossing bridge. Came into contract with D’s trolley wire and P shocked

and burned.(1) No evidence that duty was ignored. No evidence that duty was breached.(2) Ordinary caution did’t involve forethoughts of this extraordinary peril.

e) Strict Liability: doesn’t matter what you do, you will pay for the injuries if you had a duty and caused the injury, no matter how reasonable you werei) Common law: owners of animals strictly liable for injuries caused by them, whether on or off their

property. Applies only to owners who know of have reason to know that the animal in question is prone to be vicious or otherwise abnormally dangerous.

ii) Ex: Pingaro v. Rossi : P was meter reader who got bit by dog in D’s backyard while reading meter, despite her beeper having “bad dog” warning.(1) NJ’s “Dog bite” statute: owner of any dog which shall bite a person while such person is on or in a public

place, or lawfully on or in a private place, including property of owner of day, shall be liable for such damages as may be suffered by the person bitten, regardless of the former viciousness of such dog or the owner’s knowledge of such viciousness.

(2) P showed that D had breached the 3 elements of the statute and therefore was able to recover under strict liability.(a) P only needed to prove that (1) D owns dog, (2) dog bit P, and (3) P was lawfully on D’s property – b/c P

proved all 3, P wins – doesn’t matter whether D took reasonable precautions because the dog bite statute is essentially strict liability. Doesn’t matter how much care you take under strict liability scheme, you’re still liable.

f) Common carriers owe a higher duty of care than the ordinary person because of specialized skill.i) What is the highest duty of care? Not strict liability, still limited by reasonableness, but what’s reasonable has a

higher threshold. May relate to industry practices.ii) Ex: Jones v. Port Authority: Jones injured on a bus when he was getting on it began to pull away, then stopped

suddenly. Injured his arm.(1) Holding: A common carrier owes the highest duty of care (extraordinary care) and thus should be subjected

to a higher standard than the reasonable person standard. (a) Note – we also saw this higher degree of care required for pharmacy in Wal-Mart.

iii) Rationales for common carrier rule – it’s a method for risk spreading. (1) Argument against this: people should be able to choose cheap and risky. Argument against that, though, is

that poor people won’t have the benefit of safety, and everyone should be able to have that – shouldn’t get to charge more just b/c you’re safer.

(2) Common carriers pose danger to people who aren’t customers – sharing roads.(3) Also hard for people to find out information about how carefully common carrier will drive – not like your

bad driver friend you can avoid riding with – high info costs.(4) Also have issues of necessity, low bargaining power – want high duty to offset financial incentive for carrier

to not be so safe.(5) As a matter of policy, we want to encourage people to use mass transit.(6) Critics urge that conduct of common carriers be judged under ordinary reasonable person standard. p.154

(a) Common carriers aren’t sufficiently distinct from other actors(b) Heightened standard is unnecessary b/c reasonable person standard takes into account all relevant

factors

g) Campbell v. Kovich : P struck in eye by unknown, unrecoverable object that was ejected by lawn mower operated by Minish, who was mowing D’s lawn.i) Ct held that Minish was acting reasonably. He was not required to exercise extraordinary care.

Amanda Lee, Hershovitz – Torts, Fall 2009 7

Page 8: Torts outline

h) Emergencies: If D confronted with emergency and is forced to act with little time for reflection, D must merely behave as a reasonable person would if confronted with same emergency.i) Cts divided whether jury should receive a general “reasonable careful person under the circumstances”

instruction, or whether instruction should be supplemented with specifically informing jury that it may take into account the emergency in assessing the allegedly negligent party’s conduct.

i) Reckless: conscious disregard of a substantial riski) Ex: Beausoleil v. Nat’l RR Passenger Corp : Amtrak train struck person crossing tracks

2) JUDGE V. JURY, LAW V. FACT: WHO DECIDES?a) Standard of conduct, when clear, should be laid down by the court.

i) Baltimore v. Goodman (Holmes): Goodman comes up to a RR crossing reduces speed to 10-12 mph and due to building blocking his view, he fails to see a train which kills him – even though he slowed down, he didn’t see or hear train until too late. (1) Holding: RR didn’t breach duty, Goodman failed to exercise reasonable care when approaching the crossing.

He should have stopped, gotten out, and looked.(2) Because this is an event that happens frequently, the court should lay out a standard of conduct on how to

act reasonably. (a) Standard of care: driver had duty to get out of car and look if unsure if train is near.

(3) Pokora v. Wabash (1934) (overrules standard from Goodman, Cardozo calls it dicta): Ice truck driver comes up to a railroad track and doesn’t hear a train coming, so he slowly proceeds across and is hit by a train. (a) Court limits Goodman, and says that reasonable conduct in cases like this is a matter for the jury to

decide. Plaintiffs aren’t required to take unreasonable precautions, like getting out of truck to look for train (which could actually be more dangerous, b/c train could be there by the time he gets in and starts going with false confidence).

b) Rules vs. Standardsi) Holmes : it’s cheaper to have certain, rigid and specific rules – more efficient for everyone. Want rules, not jury

every time.(1) Clear rules cheaper to administrate.

ii) Cardozo : want standard, let jury decide based on the facts of individual cases – rigid rules aren’t good, because we want to avoid silly, bad and even dangerous outcomes. Too many rules can make it even harder to know obligations, so it becomes counterproductive.(1) Judges shouldn’t be making rules like what Holmes did in Goodman. (2) Consider impact custom has. (3) Political economy – want reasonable decision to be made by jury

3) THE REASONABLE PERSONa) Reasonableness of D’s conduct reviewed under objective standard: Would a reasonable person of ordinary prudence,

in D’s position, do as D did? D doesn’t escape liability merely because she intended to behave carefully or thought she was behaving carefully.

b) Objective vs. Subjectivei) Vaughan was leading early decision for proposition that issue of whether D has lived up to standard of

reasonable care is to be determined by application of objective vs subjective standard of care(1) Conduct (behavior) vs. State-of-mind (mental state): distinction between whether conduct of D was

reasonably careful (objective) vs D’s attitude was one trying to be reasonably careful (subjective) –(2) Generalized vs. Individualized: distinguish what a reasonable person would do (objective) from D’s

personal attributes (subjective) (a) Criteria against which to assess the external reasonableness of D’s conduct – compare conduct to how

an ordinary person, acting reasonably, would have behaved under the circumstances (objective) or assess conduct by considering how person with D’s attributes, acting reasonably, would have behaved (subjective)

c) A reasonable person considersi) The foreseeable risks of injury that his conduct will impose on societyii) The extent of the risks posed by his conduct.iii) The likelihood of a risk actually causing harm.iv) Alternatives to his proposed conduct that would achieve the same purpose with a different degree of risk.v) The costs of various courses of action in determining what is reasonable.

d) Time frame for assessing conduct: question is whether, in this instance, D adhered to relevant standard of care. p.170

Amanda Lee, Hershovitz – Torts, Fall 2009 8

Page 9: Torts outline

i) Grady criticism: time slice/snapshot approach introduces a pocket of strict liability into negligence law, because even the most careful person can be expected to err once in a while. No matter how careful one is, one cannot avoid sooner or later being careless.

e) Rule: Everyone is held responsible for their behavior without regard to their mental statei) Modifying the reasonable person standard to take into account mental deficiencies would result in a trial of D’s

character rather than his conduct. (1) D’s insanity does not defeat attribution of intent in torts.(2) Sudden incapacities will not defeat a finding of breach when its onset was foreseeable to actor p.169

(a) Breunig v. American Family : D hit P’s car. D’s insurance claimed that D was schizophrenic and unable to drive as a reasonably prudent person. Leading decision holding mentally disabled to objective standard.

(3) Policy factors to warrant imposition of objective standard on insane persons p.168:(a) Desirability for allocating losses b/t two innocent parties to the one who caused or occasioned the loss(b) Providing an incentive to family members and guardians of mentally disabled people to control the

behavior of those persons(c) Removing inducements for alleged tortfeasors to fake a mental disability in order to escape liability(d) Avoiding administrative problems that are created by requiring courts and juries to identify and assess

the significance of an actor’s disability(e) Forcing persons with disabilities to pay for the damage they do if they are to live active lives

ii) Vaughan v. Menlove : D negligently piled his hay, which spontaneously combusted and burnt down P’s cottages. D was really not bright – should reasonable person standard depend on a person’s intelligence?(1) Holding: When determining reasonableness of conduct, the standard of an objectively prudent man should

be used, rather than a subjective standard. Changing the standard based on intelligence would create so broad a line as eliminate reasonableness standard.(a) Look at conduct instead of mental state(b) Judge by reasonable person capacity instead of that specific person’s capacity

(2) We need a standard that defines fault in some predictable, universal way – can’t have different rule based on each D’s capacity for negligence/reasonableness.

iii) Exceptions to rule:(a) Physical disabilities: if D has physical disability, standard for negligence is what a reasonable person

with D’s physical disability would have done.(b) Children:

(i) Depending on age:1. Children under 7, not responsible for negligence at all2. Children 7-14 engaged in typical childhood activities, cts ask jury to gauge behavior compared

to “other children of same age, experience, and intelligence.”a. Exception: children 7-14 engaged in adult activities are held to the reasonable care

standard for competent adults engaging in that activity3. Children over 14, treated as adult

(ii) Tender Years Doctrine: child is incapable of negligence if less than 7 yrs old1. Vs. “Massachusetts Rule”: child found negligent if fact finder decides that child failed to exercise

a degree of care that is reasonable for similarly situated children2. Vs. § 283A of Restatement of Torts: “if actor is a child, standard of conduct to which he must

conform to avoid being negligent is that of a reasonable person of like age, intelligence and experience under like circumstances.”a. Prevailing view that in tort cases no such arbitrary limits (age) can be fixed

(iii) Ex: Appelhans v. McFall : Defendant, a five-year-old child, ran into P while riding his bicycle.1. Rule: D could not be held liable for the injuries incurred by P because he is too young – applied

tender years doctrine, even though ct disagreed – stare decisis (doctrine of precedence)2. Negligent parental supervision – ct found that holding parents strictly liable for failing to

prevent their child’s negligence was unreasonable and unsupported by law.a. Parent-child relationship doesn’t automatically render parents liable for torts of their

minor children. Parents may be liable if they don’t adequately control or supervise their childe

b. To prove claim of negligent supervision, P must show:i. Parents were aware of specific instances of prior conduct sufficient to put them on

notice that the act complained of was likely to occurii. Parent had opportunity to control the child

Amanda Lee, Hershovitz – Torts, Fall 2009 9

Page 10: Torts outline

4) CUSTOM AND ALTERNATIVESa) Custom: Courts generally allow evidence as to custom for the purpose of showing presence or absence of

reasonable care. However, this evidence is generally not conclusive. i) Fact that D has acted (or failed to act) as others in the community customarily do may provide a clue as to the

reasonableness or unreasonableness of the conduct. But the jury is still free to conclude that the industry custom is unreasonably dangerous and thus negligent. (1) Ex: The TJ Hooper : D operates a tugboat without a radio; the fact that most tugboats in the industry do not

yet have radios does not prevent the jury from holding that D’s lack of a radio was negligent. (a) TJ Hooper rule is black letter law: custom is probative, but not dispositive on the question of

reasonablenessii) Proof of custom: practice must be widespread” or “common” to count as custom

b) Medical Profession: custom as a determination of reasonableness in a malpractice settingi) Anti-TJ Hooper rule: custom dictates duty; test for doctors is whether what the doctor did was following the

custom among doctors; courts aren’t going to second guess their customs.(1) The fact that similar conduct is engaged in by those in a particular trade or profession suggests that such

conduct is acceptable – look at expert testimony.ii) Black-letter law for medical malpractice:

(1) Need an expert to explain what the custom is(2) Which doctors’ customs do we care about? (courts still disagree about which to apply)

(a) Old standard: local – was too limiting(b) New standard: “same or similar” community

(i) Most courts use “same or similar” standard because it avoids having problems of getting small town doctors to testify against one another, but also avoids problems of having standard set by a place with way more or less resources than this one

(c) National standard(i) Used to be concerned with this standard because each place might have more or less resources

than other places. But now there’s the internet, so can find out information.(3) Figure out expertise in what – the specialty of the doctor

iii) P must establish the particular standard of medical care that is required and show departure from that standard. Generally must present expert testimony.(1) P must show that D did not meet the level of a minimally qualified member of the field or specialty.

iv) Ex: Johnson v. Riverdale Anesthesia Assocs., P.C.: During surgery, anesthesia caused lack of oxygen and P died. P wanted to cross-examine D’s expert on whether he personally would’ve preoxygenated P. (1) Rule: In medical malpractice actions, the applicable standard of care is that employed by the medical

profession generally and not what one individual doctor thought was advisable and would have done under the circumstances.

v) Rationale for anti-TJ Hooper Rule (medical, legal, accounting fields):(1) Want to be more defendant-friendly – judges are elected and stand to be reelected. They don’t want to look

too patient friendly.(2) Jurors ability to second guess.(3) What these fields do are really complicated.(4) Norms of these professions are different – special responsibilities.(5) Make it harder for malpractice suits to succeed in order to keep medical costs down (careful assuming this

though, Hershovitz says there aren’t conclusive studies correlating med mal insurance costs with rising medical costs).

vi) Informed consent: doctors have a duty to disclose relevant information about benefits and risks inherent in proposed treatment, alternatives to that treatment, and the likely results if the patient remains untreated.(1) Is a limitation on per se rule for malpractice: anti-TJ Hooper rule doesn’t apply to claims of malpractice

that assert a tortuous failure to inform the patient of risks attending medical procedures.(2) Split between which standard of disclosure to use:

(a) Professional standard: physician required to make such disclosure as comports with prevailing medical standard in community; disclosure of risks that a reasonable doc would customarily make in similar circumstances

(b) Prudent patient standard (doctrine of informed consent): Require doctor to disclose what doctor should reasonably recognize would be material to patient’s decision

Amanda Lee, Hershovitz – Torts, Fall 2009 10

Page 11: Torts outline

(i) Courts want the standard to be objective – should be what a reasonable patient would want, not necessarily what X would want to know

(ii) Risk deemed material when reasonable patient would be likely to attach significance to risk in deciding whether to forego proposed therapy or to submit to it.

(iii) Causation: could establish if show that injury wouldn’t have happened because reasonable patient would’ve turned turn surgery

(iv) Critique: require disclosure of every risk(v) Reasons for adopting prudent patient standard p.181-182:

1. Existence of discernible custom reflecting medical consensus is open to serious doubt2. Doc often have to consider non-medical factors – shouldn’t be held to professional standard3. Professional standard is totally subject to whim of physicians in particular community4. Problem for patients trying to find doc willing to breach community of silence to be expert and

testify against colleagues(vi) Ex: Largey v. Rothman: D performed a surgery without informing P that there was a risk of certain

side effects of the surgery.1. Court applied prudent patient standard and ruled for P.2. Rationale: Patient has right to self-determination.

(3) Exception to informed consent doctrine: emergencies – no informed consent need to be obtained to operate on an unconscious patient in need of immediate surgery

5) REASONABLENESS, BALANCING, AND COST-BENEFIT ANALYSISa) Whether a person has behaved reasonably requires consideration of the burdens and the expected benefits of taking

precautionsi) The Hand formula (note – Rstmt 3d gives similar balancing test, but w/o specific formula)

(1) B < P x L - measures whether precautions taken were reasonable (breach)(a) (B) burden of taking the precaution(b) (P) probability that the injury will occur(c) (L) resulting loss if the injury occurs

(2) If B < P x L , it’s reasonable to take precaution(3) If B > P x L , it’s unreasonable to take precaution

ii) Hershovitz: should actually be B < P x L, so you account for the marginal difference in expected costs. iii) Hand’s description: the test doesn’t mean to quantify every possible thing, just a means of taking things into

considerationiv) Ex: United States v. Carroll Towing Co.: A barge sank when it was accidentally released from its pier, and all its

cargo was lost. If a bargee had been on board at the time, it is likely that the puncture would have been noticed.(1) Rule: To decide whether owner behaved reasonably to prevent injury, ct must consider 3 variables:

(a) The probability that the boat would break away, (P, probability)(b) the gravity of the resulting injury, and (L, injury)(c) the burden of taking adequate precautions when determining liability. (B, burden)

v) Ex: Rhode Island Hosp. Trust Nat’l Bank v. Zapata Corp.: An employee of P was forging checks and cashing them at defendant Bank. P is suing D for negligence in not detecting the forged checks. The bank will reimburse for checks paid out prior to first statement, per UCC rule, but doesn’t think it should be liable for more.(1) Holding: by following industry standards, the bank’s actions make a prima facie case of ordinary care, and

the standards aren’t unreasonable, arbitrary, or unfair because the cost of tougher standards far outweighs the benefits (a) Hand formula shows:

(i) P = very low(ii) L = $10,000-$15,000(iii) B = $125,000

(b) Since B > P x L, it’s unreasonable to take the precaution of switching check system. Procedure in place at bank was reasonable.

b) Utilitarianism: goal of social arrangements should be to maximize the aggregate level of utility or happinessi) Risk/utility balancing – if risk outweighs utility of act, risk is unreasonable and act is negligent (1st restatement)

Amanda Lee, Hershovitz – Torts, Fall 2009 11

Page 12: Torts outline

c) Posner on Hand Formula i) Formula provided basis for a more rigorous conception of balancing that asks fact finder to measure dollar cost

(DC) of precautions against the dollar value (DV) of expected harm. (1) When DC > DV, precaution shouldn’t be taken.

ii) B < P x L, evidence that law of negligence is primarily concerned to maximize aggregate social wealth by encouraging actors to take only cost-efficient precautions.

d) Professor Keating’s Criticism of Posner: moral significance that society attaches to particular activities and purposes matters in fault determination, and can’t be captured by a economic account

e) Lord Reid’s “Disproportionate Cost” testi) If, at time of action, risk that actor’s conduct would cause harm of the sort suffered by P was exceedingly small,

there was no obligation to take precautions against itii) If the risk of harm was not far-fetched – “real” – nut still very small, actor was obligated to take precautions

against the harm unless the taking of those precautions would have imposed a burden on the actor entirely disproportionate to the harm risked

iii) If risk of harm was “material” or “substantial,” the actor was obligated to do everything possible to prevent the harm, even if that would entail adopting very expensive precautions to avoid a modest expected loss.(1) Assume that he meant 1 and 2 to cover unusual cases, point 3 to cover majority of suits alleging careless

wrongdoing (2) Some said Reid’s test is better than Posner because requires actor to take precautions even when they’re

not cost-efficient. Shows that safety is more important than money or modest restrictions on libertyf) Hershovitz criticisms –Where do you cut off the expected costs? Do you have juries or academics or judges decide

the various parts of the formula? You might be able to figure out the burden, but you can’t possibly quantify the other part of the equation. Is it really the appropriate analysis for these types of disputes? i) Defense attorneys don’t want the Hand formula – see it as an inevitable way to find liability b/c juries will

overestimate risk of harm and always hold liable (i.e. Ford Pinto) – jury doesn’t ever actually get B<P x L instruction, but can have reasonableness phrased in a similar manner.

ii) Hershovitz/corrective justice view – we don’t always want B<P x L to be the sole determining factor for our obligations toward one another (more particularly, whether we’ve reasonably met the obligations that exist) – there can be big ethical quandaries(1) i.e. Ford Pinto – even if burden exceeds probable costs of tort suits, it’s morally repugnant not to fix it – can’t

measure killing people against financial cost of changing the car model – human life has more value than the tort judgment $$$.(a) But determining value of human life has always been controversial – banker 9/11 widows got more

from 9/11 fund than janitor 9/11 widows because lost wages were taken into account – people got really mad, but tort system’s valuation of a human life always includes lost wages.

6) PROVING BREACH – RES IPSA LOQUITUR (the thing speaks for itself) – classic exam mistake is to see res ipsa everywherea) In certain cases, mere fact of an accident’s occurrence raises an inference of negligence so as to establish a prima

facie case.b) Three elements of res ipsa loquitur:

i) The injury resulted from an accident that would not ordinarily occur absence of someone’s negligenceii) Must be caused by agency or instrumentality within defendant’s exclusive controliii) Must not have been due to any voluntary action or contribution on part of P.

c) Res ipsa allows a plaintiff’s case to go to the jury even though he has not proved a specific act of negligence—this is crucial.

d) When applicable in negligence actions, it permits a jury to infer that the P’s injury was caused by D’s carelessness even when P presents no evidence of particular acts or omissions on the part of D that might constitute carelessness.i) Relieves P of burden of producing evidence was to what exactly D did wrongii) Still open to D to introduce evidence to rebut the inference of carelessness

e) Once res ipsa is invoked, the burden shifts to D to prove that no negligent act occurred, because D is often in a better position to find out. D’s options:i) Prove the actual cause of the accident.ii) Attack each of the foundation facts necessary to support res ipsa.iii) When D cannot prove the exact cause of the accident, he can attempt to prove that he exercised due care in

general.f) Effect of res ipsa

i) Shifts burden of proof to D to prove facts are inconsistent with negligence.

Amanda Lee, Hershovitz – Torts, Fall 2009 12

Page 13: Torts outline

(1) Majority rule: Res ipsa gives permissible inference of negligence– jury is told it can infer negligence but is not required to, and D can testify in its defense.

(2) Minority rule (like CA): P gets rebuttable presumption, so if D doesn’t rebut, P winsg) What does res ipsa do when it applies?

i) Court normally employs 2-step process to determine breach:(1) burden of persuasion (show that more probable than not there was negligence), and (2) burden of production (show that the event actually happened).

ii) Res ipsa shifts burden of production - allows P to prove negligence through circumstantial evidence res ipsa allows jury to infer negligence based solely on what happened (shifts burden – P doesn’t have to prove negligence happens, but rather the event speaks so strongly of negligence that D must come forth with evidence that the event didn’t happen).

h) Justifications for res ipsai) In many situations, it’s more likely than not that there was negligenceii) The parties have unequal access to information.

i) Ex: Byrne v. Boadle : P was walking down the street when a barrel of flour fell from D’s window and struck P, causing injury.

(1) Rule: The accident is such that negligence on the part of D can be inferred.j) Kambat v. St. Francis Hosp .: P had a hysterectomy, and four months later, an 18x18” laparotomy pad was discovered

in her abdomen.i) Rule: There is a narrow category of factually simple medical malpractice cases that enable the jury reasonably to

conclude that the accident would not have happened without negligence.(1) Don’t need expert testimony to tell you that a pad left inside is not ordinary. Don’t need specialized medical

instruction to know – within lay knowledge.

ACTUAL CAUSE, BUT-FOR1) INTRODUCTION

a) D’s negligent act must be cause of P’s injuries in order to impose liability. Involves 2 determinations:i) Whether D’s conduct was the actual cause (cause in fact) of the injuries, ANDii) Whether it was the proximate (legal) cause

b) But-for test: If P would not have been injured but for D’s act, that act is a cause in fact of the injuryc) Two Meanings of “Caused”

i) Word conveys notions of causation and responsibilityii) Analysis concerned with how the actions of two actors happened to interact to produce a certain result

(1) Focus on determining which of the parties’, if any, brought about the injury(2) A finding of an actual causal link between actor A’s carelessness and victim V’s injury does not necessarily

entail the conclusion that A is the only person whose carelessness played a role in bringing about V’s injuryd) Preponderance of evidence: for causation, P has burden to show preponderance of evidence (more likely than not)e) Preempted causation and “doomed plaintiffs:” D can’t be held liable for injury that P already had or would have

had if D hadn’t done it.f) Why do we care about causation at all?

i) Tort law is up to deterrence (econ: inefficient conduct). Causation element plays some role in calibrating amount of deterrence.

2) BUT-FOR CAUSATION: but for D’s act, P would’ve not been injured.a) Would P have been injured if D had acted with requisite care?

i) No – but for test is satisfied and actual causation is established.ii) Yes – P’s claim fails but-for test, and usually actual causation is not established.

b) To say someone is a but-for cause, doesn’t mean they’re responsible. But-for cause is a condition necessary, but doesn’t mean you’re responsible just because you’re a but-for cause.

c) Preponderance of evidence standard: Issue is whether P has offered evidence sufficient to permit the fact finder to conclude that D’s carelessness was more probably than not a but-for cause of P’s injuryi) Ex: Skinner v. Square D. Co.: P was killed while operating a homemade tumbler with a switch made by D. P

claimed switch was defectively designed with a phantom zone, but it was merely a possibility among several possibilities, no evidence to show it was the truth – Ps have a story about how switch caused electrocution, Ds have a story about how it didn’t. (1) Holding: P’s story is plausible, but not so likely that a reasonable jury could believe that it’s more likely than

not to win on actual causation, has to be reasonable to think that it’s more likely than not that D’s act was a but-for cause of the accident.

d) Substantial factor test – CA instructs jury that carelessness on part of D is cause of an injury if it is a substantial factor in bringing about the injury.

Amanda Lee, Hershovitz – Torts, Fall 2009 13

Page 14: Torts outline

e) Can increasing the probability that harm will occur be considered a cause of the harm?i) In these cases, D’s negligence decreases P’s chances of survival, but is D’s negligence the cause of the death?

Would P have survived if D had not been negligent?ii) Many courts have ruled to treat the injury as the lost chance, and to allow the jury to value the damages in

proportion to the chance lost by D’s negligence.iii) Ex: Falcon v. Memorial Hospital: P gave birth to a child and then died from amniotic fluid embolism. The survival

rate of amniotic fluid embolism is 37.5% if the patient had an IV line in before the onset of the embolism. P was not given an intravenous line in this case. (1) Issue: 37.5% chance of survival if IV was connected. “Probable” requires 50% (more likely than not)

would’ve survived.(2) Holding: Redefining the injury to make the link between the negligent act and the result more clear - instead

of having to prove that it is more likely than not that she would have survived, P just has to prove that it is more likely than not that P would have had a better chance of survival had the proper precautions been taken. Injury = loss of chance. Falcon doesn’t change causation rule – it just redefines injury as loss of survival chance rather than focus on physical injury.(a) Case becomes over whether it is more likely then not that P lost chance to survive(b) Injury is loss of chance of survival. If IV has been there, she would’ve had 37.5% change of living. There

was certainty that would’ve had that 37.5% change of living.(i) If have rule in medical malpractice that must show more likely than not, disincentive for doc to help

people with less than 50% chance. People go to doc because they had the X% to survive. They go because they care about the small marginal difference in their health.

(3) 3 possible loss-of-chance rules:(a) Traditional – if the probability that P’s life would have been saved is greater than 50%, P recovers 100%

damages. If the probability is less than 50%, P recovers 0% damages. (b) Falcon – if the probability that P’s life would have been saved is greater than 50%, P recovers 100%

damages. If the probability is less than 50%, P recovers that % of damages.(c) Symmetrical – probability % = % of damages recovered.

(d) Problems with all three rules:(i) Traditional

1. P gets screwed if 48% of survival – no recovery2. Also, wrong incentives for doctors – as long as survival chance is at or below 50%, doctors can

act carelessly with no incentives otherwise.(ii) Falcon

1. Doctors upset because under Falcon they are paying more than under traditional – rules, taken, together, have docs paying more than the cost of injuries they inflict. If 49%, we pay 49%. If 51%, overpay and have to pay 100%. a. Doctors may end up taking inefficient precautions.

(iii) Symmetrical:1. Courts end up paying high assessment costs – big burden for jury to have to decide how likely,

1-100, that P would have survived.2. Hard system to administrate – it’s why we had traditional rule to start with.3. In civil law, we operate with what’s more likely than not – it’s efficient, making jury choose 1-

100% is too hard.(e) Is there something about medical context that warrants something besides traditional rule?

(i) Data for probabilities is pretty available(ii) Doctors have special duty inherent in medical context – people go to doctor in order to raise the

probability that they’ll survive – that’s the whole point of seeking medical treatment.(iii) Doctors assume a special responsibility.

Amanda Lee, Hershovitz – Torts, Fall 2009 14

  X>50% X<50

Traditional 100% 0%

Falcon 100% X%

Symmetrical X% X%

Page 15: Torts outline

(iv) Hershovitz view: if you construe injury as loss of chance, construe damages as reinstating the chance – bring in lottery balls to give them the chance back, rather than awarding artificial damage approximation. The only thing we know is that D is 0 or 100% responsible – we know he’s not 37.5% responsible – so why make him pay 37.5%? Just reinstate the chance and see what way luck falls.

3) MULTIPLE NECESSARY AND MULTIPLE SUFFICIENT CAUSESa) D’s carelessness need not be the only careless conduct functioning as a but-for cause of an accident to be liable.

Enough if D’s carelessness was a but-for cause.i) Carelessness of 2+ actors that brought about an injury doesn’t mean that each should be held responsible. Some

actual causes will not be deemed a proximate cause.b) Possible for P’s fault to count as one among multiple necessary causes.c) Joint and several liability: entitles a not-at-fault P to recover up to 100% of her damages from any one of 2+ D who

is found liable for the injuries. Can recover 100% from first person, 100% from second person, or a portion from each.

d) MULTIPLE NECESSARY CAUSES: if the acts of 2+ persons concur in contributing to and causing an accident, and but for such concurrence the accident would not have happened, the injured person may sue the actors jointly or severally, and recover against one or all.i) Two defendants are negligent, and both negligent acts are necessary to bring about the injury

(1) Issue of apportionment: the acts of both Ds satisfy the but-for test often, parties subject to joint and several liability.

ii) McDonald v. Robinson : Ds were negligently driving their separate cars when they collided. Their cars became interlocked, hit P, and dragged P under the car for some time.

(a) Issue: asking whether each driver is a cause, not the cause.(i) Enough if D’s carelessness was a cause, doesn’t need to be the only careless conduct.

(b) Test: could accident have happened w/o co-operation of the 2 people? There was concurrence in this case.(i) So rule: Even though Ds did not have common intent and unity of design to bring about P’s injury,

both negligent acts were but-for causes of the injury, drivers are jointly and severally liable for P’s injury.

(c) Joint liability can be established without a common intent, purpose, and design on the part of the multiple wrongdoers.

(d) EXAM SKILL: Never forget that people who caused first injury may be liable for injuries down the line.(i) Cab driver speeds around curb, doesn’t see someone’s negligently parked car, passenger hurt –

both necessary to injury, both liable.(ii) Then, passenger rushed to hospital and doctor negligently treats – both are still but-for causes of

the injury, in addition to the doctor. This doesn’t mean that they’ll be held liable for doctor’s malpractice, but they are still actual causes of it.

e) MULTIPLE SUFFICIENT CAUSES: if P sustains injury as a result of the negligent conduct of 2 tortfesors, and it appears the conduct of either one alone would have been sufficient to cause the injury, both are nevertheless liable if each of their acts was a “substantial factor” in causing the injury. i) When two defendants are negligent and either D’s negligent act would have sufficed to bring about the injuryii) Injury would’ve happened if either one acted alone

(1) Ex: Anderson v. Minneapolis: D’s fire merged with another fired, causing damage to P’s property (a) Both were sufficient causes. Can’t say “but for D’s negligence, P’s house damaged,” because there was

another fire that would’ve caused the damage.iii) Substantial factor test – explains the results when there are multiple sufficient causes

(1) An actor’s carelessness must be a substantial factor in bringing about an injury in order to be deemed a legal cause of that injury. Carelessness will be deemed a substantial factor if it constitutes:(a) A non-trivial necessary condition for the occurrence of P’s injury(b) One of two or more forces that is each sufficient to bring about harm to another multiple sufficient

cases(2) Some courts treat it as test for actual causation that permits at least some negligence P to prevail even if

can’t satisfy but-for test(3) Test actually intended for 2 different goals:

(a) To explain result in multiple sufficient cases(b) To graft onto a standard but-for test for actual causation a proximate cause restriction that would bar

the assignment of responsibility to a D who’s carelessness functions as trivial but-for cause of P’s injury

Amanda Lee, Hershovitz – Torts, Fall 2009 15

Page 16: Torts outline

(i) “Substantial factor” meant to narrow class of but-for causes by excluding insubstantial or trivial but-for causes

(4) Hershovitz criticism: courts usually misapply the test. Cause can be necessary w/o being sufficient.iv) Ex: Aldridge v. Goodyear Tire & Rubber Co.: P was an employee at a plant that got some of its chemicals from D.

P suffered illness as a result of his contact with toxic chemicals at the plant.(1) Holding: court finds that but-for causation fails (b/c of the many non-Goodyear supplied chemicals used in

the plant, Goodyr supplied only 10% of chems used), and that the substantial factor test is insufficient to help Ps because Goodyear’s conduct was insufficient alone to cause the injuries (must be sufficient before determine if substantial)(a) P could not prove that D’s negligence was a but-for cause of his injury, given the fact that many other

companies supplied chemicals to P’s plant. Only 3 of the 28 dangerous chemicals at the plant were made by Goodyear and required combining with other chemicals to actually be dangerous.

(b) P couldn’t show more likely than not that Goodyr’s chemicals caused diseases.(i) If you can’t prove that it’s more probable than not that injury wouldn’t have occurred without D’s

contribution to the problem, you can’t recover. Here, reasonable jury would probably find that the other 25 chemicals also could have caused injury.

(2) Toxic torts: allegations that victims have contracted an illness or disease after having been exposed to a carelessly or defectively manufactured product(a) It is often difficult for individual toxic tort plaintiffs to establish actual causation because medical

studies often only suggest, rather than establish as more likely than not, a general causal relationship between exposure to a particular substance and a generic disease

(b) Sometime’s there a signature illness.(c) P must convince jury that her individual illness was an instance of the general causal relationship.(d) P usually has to rely on expert witness.

(i) 2002, Rule 702 revised to reflect Daubert factors:1. If scientific, technical, or other specialized knowledge will assist the trier of the fact to

understand the evidence or to determine a fact in issue, a witness qualified as an expert by knowledge, skill ,experience, training or education, may testify thereto in the form of an opinion or otherwise, if a. the testimony is based upon sufficient facts or data,b. the testimony is the product of reliable principles and methodsc. the witness has applied the principles and methods reliably to facts of case

2. Supreme Ct extended rule to wide range of expert, not just experts opining on medical-scientific evidence

(3) Worker’s Compensation: Only bars negligence actions for workplace injuries by employees against their employers(a) Employers held strictly liable – injured worker no longer required to prove fault on part of anyone, only

that he incurred injury in course of his employment(b) Don’t receive compensatory damage awards – receive payments in accordance with certain formulae(c) Political compromise: employees were having difficulty suing their employers through tort law.

(i) Unholy trinity: 1. Fellow servant (respondeat superior): if hurt by fellow servant, can’t sue employer, can’t hold

them vicariously liable.2. Assumption of risk: modified or abandoned in most places. Employers would say that when

employees chose to work for them, assumed the risk.3. Contributory negligence: used to be that if found contributory negligent, completely barred

from recovery.(ii) Not always easy to est that employed failed to maintain reasonable safe workplace

(d) Advantages of worker’s comp:(i) Employees like it because:

1. Not subject to unholy trinity defense.2. Don’t need to show employer caused the injury. Just need the injury to occur at the workplace.

(ii) Employers like it because:1. Maximum payout is less – there’s scale that says how much money you get for each injury (cost

control – certainty of costs)a. Numbers on chart don’t reflect what jury would award. b. Charts rarely, if ever, get updated.

Amanda Lee, Hershovitz – Torts, Fall 2009 16

Page 17: Torts outline

4) CAUSATION AND BURDEN-SHIFTINGa) Rule of alternate causation: Whenever 2+ persons by their acts are possibly sole cause of harm, or when 2+ acts by

same person are possibly sole cause of harm, and P has introduced evidence that one of 2 persons/acts is culpable, then D has burden of proving that other person/act, was the sole cause of harmi) Reason for this is that practical unfairness of denying injured person redress simply because he can’t prove how

much damage each did, when it is certain that between them they did allii) Let Ds apportion it among themselves - if neither D can prove that he did not cause the injury, both will be held

liable for the injury.(1) D are ordinarily in better position to offer evidence to determine who caused injury(2) When appears from facts that only one of D could have been negligent, although P doesn’t know which one,

application of “shifting burden” rule may impose hardship on innocent D b/c he must assume the burden of proving his innocence, which he may be unable to do unless he can effectively prove the culpability of the other D.

iii) P gets benefit of joint and several liability – D is subject to liability for full amount of damagesiv) Application of but-for rule will not work since it can’t be shown which D’s fault caused the harmv) Ex: Summers v. Tice: P and 2 Ds were quail hunting. P was hit in the eye either by D1 or D2, but could not prove

which one did it.(1) Holding: When both D1 and D2 were negligent and both committed the same negligent act so that the

chances were equal that either had caused the harm, and there was no logical way to decide who had caused it, the burden shifts to each defendant to prove that he had not caused the injury. (a) Both individuals are 50% liable, but to charge damages they have to be more than 50%, ignore this and

burden shift. (2) D1 and D2 will be held jointly and severally liable.

(a) Forcing information - person who has the info is the one that did it. But in this case, Ds don’t have the info so both have to pay 50% even though only one D caused 100% of the injury.

(3) Order to think through case:(a) Each D acts in negligent manner(b) One of the Ds are clearly causally responsible(c) All that are causally responsible are in court(d) P’s innocent(e) Failure to prove isn’t P’s fault

(4) This case is different from McDonald, multiple necessary case because that case only took one of the guys to cause the injury.(a) No concert of action here in shooting because there was no mutual encouragement and plan to shoot in

that direction together(i) Court look at very narrow time slice.

b) Alternative Causation vs. Multiple Necessary Causesi) Alternative Causation: situations in which one of 2+ negligent actors acts independently of the other to cause

injury to P(1) Only one actor’s act functions as but-for cause; other has no effect

ii) Multiple Necessary: situations in which independent careless conduct of 2+ actors each functions as cause of P’s injury(1) Applies only when each of 2+ acts functions as but-for cause

iii) Conspiracy: existence of plan or undertaking by 2+ to injure Piv) Concert: treats 2 tortfeasors as acting jointly, rather than independently. Don’t need to have existence of plan to

injure P.

c) Alternative Causation Extended to Market Share Liabilityi) For P to get alternative causation, each of possible causers of her injury must be joined as a party in lawsuit;

otherwise person who actually caused harm might escape liabilityii) Even if all Ds are assumed negligent, it is uncertain which one of them actually caused P’s injury due to passage

of time and fact that D’s drugs are indistinguishable. Burden shifting doesn’t apply because there are too many tortfeasors or not all tortfeasore are before the court.

iii) In Sindell, court expanded to impose liability even in absence of all possible tortfeasors. (1) Market Share Liability Rule: if Ds in action represent a substantial share of the market for the product,

they will be held liable for a percentage of P’s injuries equal to their market share. Ds can exculpate themselves if they can show that they could have caused P’s injuries. Liability is several.(a) Each manu’s liability for an injury would be approximately equivalent to the damages caused by the

DES it manufactured; D held liable for proportion of the judgment represented by its share of the market

Amanda Lee, Hershovitz – Torts, Fall 2009 17

Page 18: Torts outline

(b) P can recover proportion of damages from each D based on the proportion they supplied. (c) Market share liability is a way of getting around an unknown D in cases where Ds are in a better

position of bearing the cost of injuries caused by their products.(d) Sindell v. Abbott Labs : P’s mother took DES while pregnant with P. P has developed adenosis and other

complications as a result of the DES. However, because P’s mother had taken the DES so long ago, she could not name the company that manufactured the product that her mother took. About 200 manufacturers of drug.(i) Difficulty in applying past doctrines:

1. Court said can’t apply Summers doctrine because need all tortfeasors in court (about 200 manu, only 11 in court). Probability of one of the 11 doing it is too small.

2. Can’t apply concert of action because there was none, even they were all doing things in common, there was no common design.

3. Can’t apply industry liability because FDA was approving drug. No trade association where safety decision was made.

4. So court used market share liabilitya. P can recover proportion of damages from each D based on the proportion they supplied. b. Burden of proof on D

i. P is not at fault for failing to provide evidence of causation – lapse of timeii. D better able to bear cost of injury – holding it liable for defects and failure to warn will

provide an incentive for product safety(ii) Difference between Summers and Sindell

1. Cause of injurya. One of D in Summers didn’t cause injuryb. Sindell – all manu caused the injury. It was a sorting problem – didn’t know which manu

caused P’s injury2. Responsibility

a. Summers had risk of holding someone responsible that didn’t do anything wrongb. Sindell, not worried about this because each manufacturer responsible for proportion

damages based on proportion distributediv) Other approaches to market share liability

(1) NY, Hymowitz case – created irrebuttable presumption of causation under which each D is liable to each P in proportion to its national market share, even if D could show that its DES couldn’t have caused a particular injury(a) Ds can’t exonerate themselves. If let Ds exonerate, messes up market share liability. Makes it so Ds

share burden of proving same thing.(2) FL, Conley case – market share calculated at geographically narrow level – state or county(3) WI, Collins case – treat Ds as joint tortfeasors, although market share is deemed relevant to jury’s

apportionment of responsibility among them.v) The effort to push market share liability beyond DES has been rejected by most courts – hard to apply. DES

cases feature unusual set of conditions rarely seen:(1) Existence of systemic reasons, not bearing on P’s diligence, for absence of evidence on product identification(2) Ability of P to bring group of D that were responsible for almost all sales of the product(3) Fact that product in questions was entirely generic and fungible in terms of design specifications, its

manufacture, its propensity to cause same illness amg diff situated victims(a) Products were all same, no difference(b) Signature illness so no remaining problems of proof

(4) Availability of at least some reliable data on market sharevi) Problems with market-share liability

(1) What market? The state where mom was at the time? City? Country?(a) NY: market is national. Avoid probs of uncertainty as to where mom was when she took pill, if she went

to pharmacy across state lines, if she moved during pregnancy, etc.(2) What procedures for allowing D to show it couldn’t have made the product (either didn’t make product at

that time, or P remembers pink pill and D only made blue)?(a) NY: can’t escape liability by showing you didn’t make the pill mom took

(i) Rationale – this evens out over time, b/c if everyone comes forward you’ll eventually pay just in proportion to your market share. Wouldn’t be fair if some company with a distinctive pill always avoided liability (or would it?)

(3) Who bears risk of insolvency of one of the drug makers?(a) So is liability joint-and-several, or just several?

Amanda Lee, Hershovitz – Torts, Fall 2009 18

Page 19: Torts outline

(i) CA: several liability – only liable for your market share. If drug manufacturer has since gone out of business, P bears that burden and not other drug manufacturers – P loses their % of market share as a % of her damages.1. This leaves Ds no incentive to help get other Ds into court, and makes it tougher for P – if she

wants money, she’d better find everybody.vii) Hershovitz says that of Summers, Falcon, and Sindell, Sindell is the smallest departure from traditional tort law

because people held responsible are all tortfeasors – the only problem is that we can’t match up right Ps and Ds.(1) In Falcon, we’re not holding people responsible for what they actually did, and may be holding them liable

for something they’re not responsible for at all.(2) In Summers, we definitely hold someone responsible who isn’t.

PROXIMATE CAUSE1) INTRODUCTION

a) After show actual cause, must show proximate cause.i) Proximate cause only arises in cases where actual cause is proven or presumed.

b) Must draw line between immediate and remote causes proximate causec) Question to ask: whether there is something about the causal connection between D’s carelessness and P’s injury

that warrants freeing D from responsibility for that injury notwithstanding that connect.d) Question for the jury.

2) FORESEEABILITY: D is liable for consequences of his negligence that was reasonably foreseeable at the time he acted.a) Old tests – no longer in use

i) “Natural and ordinary”/”one leap” rule – (no liability for certain foreseeable results)(1) Ryan v. NY Central RR Co (1866): RR engines sparked and ignited fire in shed that spread to P’s house.

(a) No liability for certain foreseeable results - ct said D was not proximate cause because it was neither natural nor ordinary for fire to spread further beyond one other structure.

ii) Directness test – (liability for unforeseeable consequences)(1) In Re Polemis (1921): D leased a steamship from P. D’s employees accidentally knocked a wooden plank into

the hold, which started a fire when it hit the flammable benzene vapors. The fire destroyed the ship.(a) Liability for unforeseeable consequences – ct held that D was liable for all direct consequences of his

wrongful conduct, despite the occurrence of an unforeseeable type of injury.(i) D liable for harm he has directly caused; fact that the actual risk created by D differed from that

reasonably to be anticipated was deemed immaterial.(2) Problems with the direct cause test

(a) Does not explain the results in real cases. (b) Under-inclusive: superseding conduct always precludes liability of D; however, there are many cases in

which D should be held liable despite intervening forces.(c) Over-inclusive: there are some cases in which D was directly responsible for the harm, but we do not

want to hold him liable because the injury that occurred was so freakish and unforeseeable. b) Current – FORESEEABILITY TEST – (no liability for unforeseeable consequences)

i) D is liable if, at the time he acted, he should have foreseen the risk that injured P. No liability for unforeseeable consequences.

ii) Provides a more solid analytical basis for consistent decision-making—the judge or jury can ask what reasonable risks D should have anticipated at the time he acted, and then compare those risks to the injury that actually occurred.

iii) What has to be foreseeable?(1) Harm or type of harm needs to be foreseeable.(2) Harm to that class of persons.

iv) What doesn’t need to be foreseeable?(1) Precise way harm comes about(2) Don’t need to foresee extent of injury

v) Ex: Wagon Mound I (1961): A ship’s employees spilled oil into the harbor, which ignited and destroyed a dock and another ship nearby. Demise of Polemis directness test.(1) Holding: The employees on the ship were not liable for the damages because it was unforeseeable that the

oil would catch fire due to sparks from welding.(2) Ct found that Polemis directness test not good law:

(a) Man must be responsible for probably consequences of his acts(b) That D could foresee property damage in one form was not sufficient to hold it responsible for an

entirely different sort of damage.vi) Ex: Wagon Mound II

Amanda Lee, Hershovitz – Torts, Fall 2009 19

Page 20: Torts outline

(1) Ct held that fire that destroyed dock was a foreseeable consequence of furnace oil spill, and thus should be deemed a proximate cause.

vii) Union Pump Co v. Allbritton : A faulty pump caused a fire at a chemical plant. P stayed at work longer than usual because of the fire, and later went to fix valve (following boss taking shortcut over some pipes rather than a longer, safer way). P got injured on the way back, taking the shorter way.(1) Holding: Pump fire did not more than ceate the condition that made P’s injuries possible. Circumstances

surround her injuries were too remotely connected with D’s conduct or pump to be proximate cause.(a) Pump defect was “but for” cause of her injuries. But not foreseeable that injury would’ve happened.

c) Risk Rule – Prof Keeton, leading present-day alternative to foreseeability formulationi) A careless actor is subject to liability for harm actually caused by his careless conduct if the harm “is a result

within the scope of the risks by reason of which the actor is found to be negligent.”(1) Is the injury that occurred among one of the risks that led us to classify that type of behavior as negligent in

the first place?(2) Is injury a realization of risk that made D’s conduct negligent?

ii) Third Restatement adopts a variation of the risk rule: whether the injury suffered by the victim is one of the harms whose risks rendered the actor’s conduct careless.

iii) Ex: Jolley v. Sutton: P, school boy, injured while repairing and painting boat which fell on top of him. Boat was derelict and rotten, supposed to have been removed by D – had placed warning sticker on boat.(1) Ct held that P must show that injury which he suffered fell w/in scope of D’s duty. Scope of duty determined

by whether or not injury fell within a description which could be said to have been reasonably foreseeable.(a) Occupier’s Liability Act – common duty of care: duty to take care as is reasonable to see that the

visitor will be reasonably safe in using the premises for the purposes for which he is invited or permitted by the occupier to be there.(i) Occupier must be prepared for children to be less careful than adults.

(b) D was under a duty to protect a child from a danger by taking reasonable steps.(2) Boat was a trap/allurement. D should’ve removed it. It was foreseeable that children would want to play

with boat. P’s form of play was mimicking adult behavior. Accident and injury were foreseeable.(3) Was injury to P a realization of risk that made D leaving boat behind negligent?

d) EXAM TIP: EXAMINE CASE USING BOTH FORESEEABILITY AND RISK RULE APPROACHe) Hyptheticals on p.285 – do them!

3) SUPERSEDING CAUSEa) Does the injurious acts of an intervening wrongdoer function to block an attribution of responsibility to another

wrongdoer whose wrongdoing also helped bring about the injury?b) To be superseding cause, intervening act must be so “highly extraordinary” that antecedent negligence should be

ruled out as a matter of law as a substantial factor in causing the accident.c) A third person’s negligent conduct doesn’t relieve D of liability – even though conduct was unforeseeable – if

it causes a result similar to that threatened by D’s conduct, unless act was “highly extraordinary” under the circumstances.i) If criminal act is foreseeable, then that act doesn’t cut off the negligence of the other.

d) Britton v. Wooten : P is suing her lessee because he let his trash pile up, which was lit on fire by an arsonist and burned down the property.i) How fire started doesn’t matter – issue is whether P can prove that D caused/permitted trash to pile up next to

its building in a negligent manner which caused/contributed to spread of fire and destruction of building. If so, fire is not superseding cause.

ii) D argued that when someone commits a crime, it’s a superseding cause. Ct says not the rule anymore – criminal conduct is not a superseding cause. Superseding cause should be where act is highly extraordinary.

4) PALSGRAF: P standing on plaform when D’s train pulled into station. 2 men rush forward – one had pkg with fireworks tried to jump aboard with help of conductors. Pkg exploded, causing scaled at other end of platform to fall and injure P.a) Famous case because Cardozo and Andrews communicate how tort law works in their viewsb) Cardozo view – duty owed only to “foreseeable plaintiffs”

i) Palsgraf’s issue is about duty.ii) D owes a duty of care only to those persons as to whom the reasonable person would have foreseen a risk of

harm under the circumstances. Must appear that the reasonable person would have foreseen a risk of harm to P or a class of person to which P belongs – duty is relational.

iii) In Palsgraf, this means D owed no duty to P – a reasonable person would not have foreseen any risk to P. Conductor’s negligence wouldn’t forseeably cause injury. Duty breached by D wasn’t the duty owed to P. D had a duty to man with package.

c) Andrews view (dissent) – if duty owed to anyone, duty owed to all

Amanda Lee, Hershovitz – Torts, Fall 2009 20

Page 21: Torts outline

i) Palsgraf’s issue is about proximate cause.ii) D’s duty of care is owed to anyone who suffers injuries as a proximate result of D’s breach of duty. Where P is

injured by D’s negligent act, D is responsible regardless of whether reasonable person would have foreseen a risk of harm, as long as P’s injuries were proximately caused by D’s negligence toward someone.

iii) Andrews – duty to the world, not relational.(1) But there’s policy concern of how far we want liability to extend. Policy concerns force us to limit those who

recover from breach of duty to those who would foreseeably be injured.iv) In Palsgraf, this means that D owed P a duty of due care – duty owed to everyone. Issue is proximate cause – P’s

injury was actually caused by D’s negligence. Not just all about foreseeability – you have a duty all the time.(1) No policy reason to cut off negligence.

d) IDEOLOGY: Cardozo and Andrewsi) Cardozo – tort law is about redressing wrongs to people – “there’s no negligence in the air,” all that’s wrong is

wrong to a person. Tort system isn’t about regulating risks, it’s a private system for redressing wrongs with respect to you.(1) This is why we need a tighter connection between duty and breach, and why duty is relational.

ii) Andrews – tort law is about enforcing standards for how we want people to behave.(1) If tort law is about setting up guidelines for behavior, we want an expansive view of duty, and don’t need a

tight connection between duty and breach. If it’s really unfair to impose liability, public policy/justice will step in through proximate cause and cut it off.

e) Hershovitz thinks Cardozo has the better opinion on duty being relational. Risk rule or foreseeability should be for proximate cause in modern rule. Hershovitz is inclined to put the policy question with duty, like the CA/NY courts.

f) “Relational” aspect of negligence lawi) Bourhill v. Young: D struck and killed in accident. P leaving train saw blood and was traumatized.

(1) Ct said D wasn’t negligent because P couldn’t show that in relation to her, D acted negligently.g) Look at other outline and text for more notes

STATUTORY SUPPLEMENTS: NEGLIGENCE PER SE, WRONGFUL DEATHS1) NEGLIGENCE PER SE: used to establish breach; incorporates standard of behavior contained in criminal/regulatory

statutes that, on their face, don’t say anything about tort liability, but sets up a standard of carea) General

i) Legislatures often enact standards that establish standards of care for common situations.(1) People injured due to a violation of such a statute usually claim that D was negligent for failing to comply

with the statutory standard of care.ii) Where a duty of care is already owed, and a statues provides that specific conduct breaching that duty is

subject to criminal penalties, under appropriate circumstances, courts may use violation of the criminal statutes to establish a breach of duty.(1) Advantage is that if P can use negligence per se, don’t need to prove breach.(2) Issue of breach: permits P to satisfy her cause of action by proving that D violated a certain kind of statutory

rule of conduct(a) Relieves P of her burden of proving that D violated common law’s reasonable person standard.

iii) Negligence per se doctrine only applies to violations of statues that are intended to set standard of conduct, versus statutes that serve as record keeping or other administrative functions (like licenses)(1) Exception: Medical licenses can qualify for negligence per se

iv) If court finds that P can’t use negligence per se, it still has to undertake the ordinary common law negligence analysis. Although P’s prospects of prevailing are poor.

v) Some courts admit statutory violations as evidence of negligence versus negligence per se.b) Requirement for negligence per se:

i) Statute must be clear and unambiguous. Must specify exactly what conduct/duty is required, of whom it is required, and what constitutes a breach of the duty. Safety oriented statute.

ii) Must also appear that in enacting the statute, the legislature was seeking to accomplish 2 objectives:(1) To prevent the particular type of injury involve in the current tort action, and(2) To protect the particular class of P involved in the current tort action

Amanda Lee, Hershovitz – Torts, Fall 2009 21

Cardozo Andrews Androzo 1 Androzo 2Duty Duty is relational Duty isn’t relational Duty isn’t relational (CA and NY law)

Duty is not relational, but policy considerations factor in

Proximate Cause Polemis directness view Proximate cause is a policy question

Risk rule Foreseeability or risk rule

Page 22: Torts outline

(a) Who’s the protected class?(i) Several courts have been confronted with questions of whether workplace safety regulations are

for only employees or also independent contractors, and invitees of business.iii) Statute can be invoked to set a specific duty of care only if D has no legally acceptable excuse for its violation.

(1) Sometimes D’s violation is reasonable.c) D’s proof of regulatory compliance

i) If D establishes that her conduct complied with requirements of statute, doesn’t mean conduct was reasonable compliance isn’t dispositive of reasonableness.

ii) Compliance is typically treated as evidence of reasonablenessiii) In many jurisdictions, compliance can defeat or limit D’s liability

d) The Right Sort of Injuringi) Negligence per se doesn’t apply when:

(1) Chevron v. Forbes : P slipped in puddle of gas at gas station. Couldn’t do neg per se b/c rule for attendants to clean up spills was for fire-safety measures.

(2) Wawnesa Muts v . Matlock : P sued D that had provided cigarettes to minor who had dropped it on lumber and cause P’s lumber to set on fire. Ct said can’t use neg per se b/c rule of not providing cigarettes to minors was to prevent young persons from becoming addicted to cigarettes.

(3) Gorris v . Scott : P sued for D failing to keep sheep in separate pens in ship, violating statute. Some sheep were washed overboard. Can’t apply because ct rule purpose of statute was to prevent spread of infections.

ii) Negligence per se applies:(1) De Haen v . Rockwood Sprinkler : P killed by D’s worker when radiator fell on P b/c hoistway didn’t have

full enclosure. Ct ruled could use neg per se b/c statute enacted to prevent workmen from falling and objects from falling.

e) Case examples:i) Dalal v. City of New York : P was hit in an intersection by D, who wasn’t wearing his glasses at the time of the

accident, in violation of a restriction placed on his license. (1) Holding: There is negligence per se here, because the statute was designed for exactly this purpose.

(a) Violating restriction on license wasn’t a violation of a licensing statute – it was a violation of a regulation about the manner in which D drives the car – key to understanding why negligence per se can attach here, but not for driving w/o a license.

ii) Bayne v. Todd Shipyards Corp.: P fell from a loading platform while on D’s premises, while making a delivery to D. P wasn’t an employee of D; was employed by the trucking company. D violated an administrative regulation by not putting guardrails on his loading platform.

(a) Issue: is violation negligence per se or only evidence of negligence?(i) Court held that violation was negligence per se because it was created by Dept. of Labor and

Industries, to make the workplace safer for workmen.(ii) Dissent would consider administrative regulations evidence, but not negligence per se.

1. Administrative statutes are unnecessary and impractical, so many are churned out. a. But legislative statutes should be given more weight.

2. Better for violations to be submitted as evidence of negligence, trier of fact has better opportunity to use common sense and reach a more nearly just result b/t the parties.

(b) Negligence per se questions to ask:(i) Is P within the class of people meant to be protected?(ii) Was the injury P sustained like the kind that the statute was meant to prevent?

1. Yes. Because the statute was meant to make workplace safer for everyone, it even applies here to a non-employee – he was within class of people intended to benefit from the administrative safety regulation.

iii) Victor v. Hedges : D parked his car on the sidewalk, in violation of a statute, in order to show P his new CD player. A third party lost control of his car, ran the curb, and hit P, pinning her up against the parked car.

(a) Holding: The statute making it illegal to park cars on sidewalks was not designed to prevent being struck on the sidewalk by a vehicle other than the illegally parked vehicle. It was instead meant to prevent obstruction of pedestrian traffic on sidewalks and injury to pedestrians due to a car parked on a sidewalk. Therefore, D’s violation of the statute is not negligence per se (statute not designed to protect against this type of hazard).

(b) This is like proximate cause and risk rule analysis: is the risk that materialized during the accident the same risk that led the legislature to enact the statute in the first place?

f) Excused Violations

Amanda Lee, Hershovitz – Torts, Fall 2009 22

Page 23: Torts outline

i) Some actors who have violated statutes are nonetheless excused from being held per se at fault:(1) Young children(2) Some cts will excuse violations where evidence that violation was more prudent course of conduct for D to

follow(3) Some cts will excuse violations if D can show that it was unable despite reasonable diligence to comply.

ii) Restatement § 288A, comment j: it is for the court to determine if excuse proffered by statutory violator is of a type recognized by law of negligence, and for jury to decide any relevant factual disputes as to whether the conditions necessary to establish the excuse have been made out

b) Arguments for negligence per sei) When a legislature has made a law saying that certain precautions should be taken, a D who violates the statute

has ignored the standard of care established by the legislature. Reasonable people don’t do this.ii) If the jury is permitted to find that D acted with due care, despite his violation of a statutory standard of care,

the jury is being licensed to disregard the command of the legislature.iii) Shouldn’t the standard of conduct enforced by the courts be the same as that established by the legislature?iv) It creates uniformity and legislatures are designed to set up rules.

c) Arguments against negligence per sei) In unusual circumstances, it may be reasonable to disregard a statute.ii) In other circumstances, it may be impossible to obey the law.iii) Imposes a large burden on the general public to follow every single statute out there, even the unreasonable

ones.iv) Most statutes that establish standards of care do not provide that a violation of the statutory standard of care

automatically establishes negligence.v) It is doubtful that the legislature intended blind adherence to statutory standards regardless of the

circumstances.

2) WRONGFUL DEATH ACTS: statues that confer on P power that Palsgraf denies – power to sue as “vicarious beneficiaries” of rights possessed by othersa) Survival actions: brought by estate of person who died; damages recoverable provide decedent’s estate with

compensation for any harm the decedent suffered up to the moment of her death.b) Wrongful death acts

i) “Wrongful”: if conduct would have amounted to a tort had the victim been injured, it constitutes “wrongful” death when death rather than injury results

ii) Concern was to compensate (1) widowers who had lost valuable services and (2) widows and orphans who had lost their breadwinner

iii) Compensates immediate family members for the losses that they have suffered because of the decedent’s death – this is why claims are “vicarious” or “derivative” in nature(1) Value of damages: loss of future support they would’ve gotten from the person who died(2) Not being compensated for grief

c) Loss of consortiumi) Courts are split on whether parents can bring loss of consortium actions for death of a child.

(1) If yes, damages are very small.d) Wrongful death and loss of consortium actions are derivative claims:

i) Claims in survival and wrongful death actions are limited by P’s claims. (1) If P’s recovery would be limited under a comparative fault scheme, both survival and wrongful death

actions are reduced by the same percentage. (2) If P’s recovery were barred by statute of limitations, neither survival nor wrongful death actions could

survive.

DEFENSES1) CONTRIBUTORY NEGLIGENCE

a) Old common law rule that completely barred recovery if P was slightly negligent in cause of her injuryb) Rationale:

i) Wanted incentive for people to take care of themselvesii) Tort law not in the business of helping people who behave badly

c) Doesn’t extend to intentional tortsd) Harshness of contributory negligence ameliorated by inform practice (ad hoc adjustments by jury) and doctrine of

last clear chancei) If D has the last opportunity to prevent an accident resulting from careless acts of both D and P, D will not enjoy

the protection of the contributory negligence defense.

Amanda Lee, Hershovitz – Torts, Fall 2009 23

Page 24: Torts outline

ii) P bears the burden of proving the doctrine’s applicability. e) Elimination of contributory negligence replaced by comparative fault

2) COMPARATIVE FAULTa) Comparative fault rejects rule that negligence claim must be dismissed upon finding that some carelessness on part

of P contributed to her injuryb) Under the doctrine of comparative responsibility, D must prove causation between P’s negligence and P’s injury, just

as P must prove causation between D’s negligence and P’s injury. c) Pure system

i) Most jurisdictions, P may recover, regardless of degree of fault, but her recovery is reduced by her % of fault. (1) Ex: So if P is 60% at fault, she could recover 40% in damages

ii) Criticism: is it fair to pay damages to one who is the overwhelming cause of his own misfortune?iii) Ex: United States v. Reliable Transfer Co.: P’s tanker was beached on a sandbar. D, coast guard, was negligent in

failing to maintain a light, and P was negligent in failing to use available equipment in setting a course.(1) Court found that P was 75% at fault, and D was 25% at fault, and held that liability for damages is allocated

among parties proportionately to the comparative degree of their fault. (2) This broke from divided damages rule, which would have divided recoveries up evenly in proportion to how

many people contributed to the accident, regardless of each party’s level of contribution(a) 2 parties: 50-50. 4 parties: 25-25-25-25, even if actual percentage of fault is 10-10-20-60.

d) Modified comparative fault: Recovery denied if P’s fault passes a certain threshold (usu 50% - if P’s fault is equal or greater than fault of D)i) Not-as-great-as form: P can recover only if her % fault is below that of D. If there are two parties, this means P

must be below 50%. ii) Not-greater-than form: P can recover when P’s and D’s negligence is equal. Because juries often conclude that

parties are equally at fault, this difference has quite an impact.iii) Multiple defendants: P barred from suing if her comparative fault is equal or greater to the total fault of all

those whom she is suing(1) Some juris, P’s negligence must be less than that of any defendant

iv) Some states on instructions of jury, inform jury of legal effects that will follow from assignations of percentage fault. (1) However, concern that juror will adjust calculation of percentage fault to avoid seemingly harsh legal

consequences that would follow from “true” assignment of percentagesv) Criticisms:

(1) a P that is 51 percent at fault will not recover anything, and will have to pay 49 percent of the damages to a D who counterclaims.

(2) Hershovitz criticism of comparative fault in general: asking a jury to put a % on fault is incredibly difficult, if possible at all.(a) Oftentimes it’s either just enough to recovery or just enough to not recover!

vi) Ex: Hunt v. Ohio Dept. of Rehabilitation & Correction: P, an inmate in custody of D, was operating a snowblower at the request of D, when she stuck her hand down the chute to unclog it. Several fingers were severed.(1) OH comparative negligence rule bars P from recovery if his actions were a greater cause (50+%) of his

injuries than acts of D.(a) P was 40% responsible, so claim not barred.(b) Damages were $18,000. Reduce by 40% $10,800.

vii) Baldwin v. City of Omaha : P was mentally ill. After discharged from hospital has psychotic episode where 2 police offices attempted to subdue him – one shot him leaving him paralyzed from chest down(1) Nebraska rule had changed from contributory neg to modified comparative fault, which stated that if

claimant’s fault contribute to his injury to an equal or greater degree than fault of all persons against whom he seeks recovery, recovery barred.

(2) Trial ct round that 55% Baldwin’s fault for not taking medication and 45% officers’ fault for acting unreasonable in detaining mentally ill suspect(a) --> Meant P was barred from recovery. Result was the same under the old and new rule.

3) ASSUMPTION OF RISK: When P knowingly and voluntarily takes the risk that she might be injured by careless conduct on the part of D.a) Required elements:

i) P recognized and understood the particular risk or danger involvedii) P voluntarily chose to encounter itiii) Then - Are there policy reasons for courts to decline to enforce such assumptions of risk?

b) Two types:

Amanda Lee, Hershovitz – Torts, Fall 2009 24

Page 25: Torts outline

i) Express assumption of risk: waiver in contract - P who is injured by negligence of another can sometimes be barred from suing b/c she has formally ceded or abandoned the right that tort law would otherwise confer to her

ii) Implied assumption of risk: P barred from suing for negligence by another b/c she was aware of a discrete and significant risk of injury posed to her by carelessness on part of the other, yet freely chose to proceed so as to expose herself to that risk

c) EXPRESS ASSUMPTION OF RISKi) Exculpatory clauses – where relationship b/t P and D arises out of contract, D may attempt to limit or exclude

liability. Whether P is barred or limited by assumption of risk depends on enforceability of the provisions as a matter of contract law.(1) Contract void as a matter of public policy if:

(a) Contract is an adhesion contract, where drafted unilaterally and forced upon an unwilling and unknowing public for services that can’t be readily obtained elsewhere (i) Must show that there was:

1. Disparate bargaining power, or2. No opportunity for negotiation, or3. Service couldn’t be obtained elsewhere

(2) Validity of exculpatory provisions of contract – 4 factors(a) Existence of a duty to the public(b) The nature of the service performed(c) Whether the contract was fairly entered into(d) Whether the intention of the parties is expressed in clear and unambiguous language

(3) Importance of adequate font size and P having the option to pay more and reserve his right to sue for negligently caused injuries

(4) Wavier is never valid against intentional, reckless, or willful/wanton conduct.ii) Arguments for enforcing exculpatory contract:

(1) Give places an incentive to stay open and/or offer services(2) Keep the costs of activities down.

iii) Arguments for invalidating exculpatory contract:(1) They remove the incentive to maintain safe facilities(2) Policy problems with public assuming the entire cost – facility may be better able to risk-spread and cost-

spread.iv) Cases:

(1) Jones v. Dressel : P signed a contract that included an exculpatory clause that allowed him to use D’s recreational skydiving facilities. P was injured as a result of a plane crash while skydiving with D.(a) Holding: The exculpatory clause in a skydiving contract was upheld – no adhesion contract and

exculpatory agreement was valid. (2) Dalury v. S-K-I, Ltd .: P signed an exculpatory agreement before skiing on D’s premises, and then was injured

as a result of running into a poorly placed metal pole.(a) Holding: A release form at a ski resort is a contract made in the public interest – exculpatory contract is

invalid(i) It sells a substantial number of tickets as a result of its general invitation to the public to utilize its

facilities legitimate public interest arises (b) If D permitted to have broad waivers of liability, an imp’t incentive for ski areas to manage risk would

be removed.(c) D might be able to risk spread better, could build the risk into the price of the ticket.

d) IMPLIED ASSUMPTION OF RISK: P barred from suing for negligence by another b/c she was aware of a discrete and significant risk of injury posed to her by carelessness on part of the other, yet freely chose to proceed so as to expose herself to that risk – genuinely knowing and voluntary decision

e) Primary implied assumption: a P that chooses to participate in an activity that has an inherent risk is owed no duty by other participants (ex: sports) – voluntary exposure to an obvious and known riski) Question of law for ct vs question of fact for juryii) Does not concern whether P actually appreciated and chose to encounter relevant dangers. Even if P didn’t

know of dangers or sought not to be subjected to them, primary can still applyiii) Avila v. Citrus Community College : Baseball case in which the court ruled that getting hit by a pitch is a part of

the game, even though it’s against the rules. (1) Holding: Implied assumption of risk – Avila should have known that he was signing on to play a dangerous

game.

Amanda Lee, Hershovitz – Torts, Fall 2009 25

Page 26: Torts outline

f) Secondary implied assumption: comparative treatment of D and P’s fault; can’t call this comparative fault because you’ve behaved reasonably, but your choice to confront the risk will bar you from claim i) Ex: Smollet v. Skayting Dev. Corp: P was an experienced skater that while skating at a rink, to avoid child,

swerved to carpeted area where fell and broke her wrist.(1) Ct ruled for D – evidence that P fully understood the risk of harm to herself and voluntarily chose to enter

the area – she implicitly assumed risk of injuryg) Difference between assumption of risk and negligence:

i) Assumption of risk involves seeing a danger and reasonably deciding to take the chance – mental state is imp’t(1) Bars full recovery

ii) Negligence: looking at whether behavior was unreasonable, mental state is not imp’t(1) Comparative fault – jury can decide P behaved unreasonably and reduce recovery

h) Comparative fault and implied assumption of riski) In Smollet, Virgin Island’s adoption of comparative fault raises question of viability of implied assumption of risk

defense(1) Some argue should eliminate every all-or-nothing P conduct defense(2) Smollet, Ct of Appeals took middle position – adoption of comparative fault eliminated implied assumption

of risk as a defense for any instance in which P’s conduct could plausibly be described as both unreasonable and a voluntary encountering of a known danger(a) Odd effect arises: treats reasonable P more harshly then unreasonable P(b) Some jurisdictions, to avoid odd effect, have ruled that adoption of comparative fault has completely

eliminated implied assumption of a risk as a distinct defense(i) These juris haven’t eliminated express assumption of risk

ii) To avoid Virgin Island’s problem, many jurisdictions have ruled that the adoption of comparative fault completely eliminated implied assumption of risk

iii) NY’s “Hybrid Statute: NY has retained implied assumption of risk, yet at the same time has converted it into a partial vs. complete defense(1) P’s damages shall be diminished in proportion which culpable conduct attributable to claimant bears to

culpable conduct which caused damages

4) STATUTES OF LIMITATIONS AND REPOSEa) Statute of Limitations

i) General(1) Starts clock running in relations to occurrence of (a) the alleged tortious conduct and (b) harm to the

claimant caused by that conduct. (a) Will specify that a tort claimant must commence her lawsuit within a certain pd of time.

(2) One way to cut down on tort claims is to shorten SOL(a) Personal injury, 6 months-4 yrs

ii) Accrual Rule: clock on negligence claim began to run as soon as:(1) D acted carelessly and (2) that act caused some harm to P. (3) Fact that P didn’t know, and perhaps couldn’t know of harm or its connection to D, deemed irrelevant.

iii) Discovery Rule: stalls commencement of limitations clock until:(1) P knows, or has reason to know, that she has suffered an injury, or (2) there’s sufficient reason to believe that Ds conduct is causally linked to that injury such that an inquiry into

connection is warranted and would reveal evidence of such connectionb) Statutes of repose : Set limits by reference either to the date of the tortuous act alone, or to some other date.

i) Will say that no tort action may be brought more than X years after the date of the initial sale, etc.ii) Attached to products, longer time frame, 10-15 years

(1) Special interest groups want length shorter

5) IMMUNITIESa) Charitable immunity: some charitable organizations have immunity from tort liabilityb) Sovereign immunity: the government is not subject to liability unless it consents to be sued

i) Tort Claims Act: Sets out the circumstances that you can sue the governmentii) West Fall Act: Immunizes government employees from being sued, but substitutes government as defendant

(1) Gov’t has immunities that defendant didn’t

Amanda Lee, Hershovitz – Torts, Fall 2009 26

Page 27: Torts outline

PART 2: DAMAGES1) COMPENSATORY DAMAGES

a) Goal: to restore P insofar as possible to her condition before she was injured, status quo anteb) Elements:

i) Economic/out-of-pocket losses for past and future losses(1) Medical expenses (2) Lost earning in past(3) Lost earning capacity – future earning potential(4) Reproducing Inequality? Experts who testify to lost future earnings generally rely on statistics as to how

much a person with same education, background, traits could have expected to earnii) Non-economic losses for past and future losses – difficult to quantify

(1) Pain and suffering — physical pain and mental suffering as a result of the injury, both past and present. (2) Loss of enjoyment of life — loss of the opportunity to engage in many of life’s pleasurable activities, both

past and present. Can include loss of previous lifestyle.(a) Courts are split over whether loss of enjoyment of life is a type of pain and suffering, or a distinct

category of damages.c) Normally paid out in lump sum vs. structured settlement payments

i) Lump sum – pay at once(1) Problems:

(a) If jury under or overestimated, no recourse for P(b) Calculation of inflation over time

ii) Structured settlements – pay out over time(1) Help P deal with sudden windfall(2) Help defendant deal with adjustments for market and economic conditions over time – better at handling

uncertain futures.d) Eggshell skull rule: tortfeasor can’t complain that amount of damage caused to other was much greater than

anyone could reasonably have expected b/c of hidden physical vulnerability in P – doesn’t matter if P does or doesn’t know that he has vulnerabilityi) No eggshell property rule – no law for if the extent of damage to property is greater than expected

(1) If you knew if was weak, do something about it yourself, fix it.e) Bifurcation – trial cts can bifurcate trial b/t liability and damages; P usually resist bifurcation

i) Concern that juror’s judgments as to issue of liability will be swayed in favor of P b/c of their visceral responses to such evidence

f) Judge v. Jury: Black-letter doctrine holds that a judge shouldn’t second-guess a jury’s compensatory damage award, unless after viewing evidence in light most favorable to P, verdict shocks the conscience.i) Remittitur: Gives P option of accepting a lower damage award in lieu of a new trialii) Additur: Gives D option of paying larger award in lieu of new trial

g) Comparison with other awards:i) Some jurisdictions allow trial judges to gauge the alleged excessiveness of a jury award by comparing it with

awards given in other similar cases;ii) Other jurisdictions don’t allow comparison b/c each case stands on its own

h) Collateral Source Rule: tortfeasor isn’t entitled to present evidence at trial indicating that victim has received, or stands to receive, compensation for her injuries from some other sourcei) About ½ state have modified/abolished rule

i) Injunctive relief: ct ordering D to cease engaging to a particular activityj) Mitigation of damages: P has a duty to mitigate damages. D may argue that he is not responsible to compensate for

certain damages flowing from his negligence because P could and should have taken certain steps to avoid them, and didn’t.i) Ex: P refuses to obtain post-accident medical treatment, and as a result injuries are exacerbated.

k) Cases:i) Smith v. Leech Brain & Co. Ltd . : P was splashed in the lip with hot molten metal and subsequently developed

cancer in the wound. Developed cancer at the point of wound and died.(1) Ct rules for P because of thin skull rule, D must take P as they find him.(2) Wagon Mound: distinction b/t question of whether a man could reasonably anticipate a type of injury, and

question of whether a man could reasonably anticipate the extent of injury of the type which could be foreseen.

(3) Ct sees that D could’ve reasonably foreseen the burn, but couldn’t foresee the cancer – should D pay for foreseeable injuries, or pay for all resulting injuries?

Amanda Lee, Hershovitz – Torts, Fall 2009 27

Page 28: Torts outline

(4) Could be described as:(a) Type of injury and extent of an injury - Thin Skull rule(b) One type of injury and another type of injury – Proximate cause

(5) Question is whether D could reasonably foresee type of injury P suffered. The amount of damage which P suffers as a result of that burn, depends upon the characteristics and constitution of the victim.(a) Damages reflected that P might’ve developed cancer even if he hadn’t been burned.

ii) Kenton v. Hyatt Hotels Corp. (1985): P was severely injured when two skywalks in the lobby of D’s hotel collapsed onto the lobby floor. As a result, unable to return to law school or to practice law.(1) Rule: In determining compensatory damages, the jury is free to hear detailed evidence about the accident

scene and about P’s potential future employability in order to evaluate P’s physical and mental injuries. P was arguing that she suffered from post-traumatic stress syndrome, so it was appropriate to admit evidence about the event of the day in question.

(2) Calculation of economic loss is amount of $ person would’ve made depending on their career choice – evidence that P’s cost of therapy would increase over her lifetime, and her income would also increase if she were a lawyer.

l) What is the purpose of torts?i) “Make P whole again” – doesn’t really happen. Juries are asked what’s fair and reasonable compensation.ii) Social insuranceiii) Securityiv) 3 purposes :

(1) Giving compensation is to bribe P to come forward and complain of D’s actions (internalize externalities)(2) Morally right – P’s entitled to money(3) Contributive negligence/comparative fault – if P doesn’t get money, doesn’t have incentive to take care –

might result in violence.(a) Torts is about providing a civil means for recourse

v) For damages, a jury wants to:(1) Make D pay enough so as to deter other Ds in the future(2) Give P enough inducement for them to take care

vi) Two views :(1) Corrective justice: backwards looking, looking at the loss P suffered(2) Economist view: forward looking, doesn’t care about P’s loss (sunk costs). Wants to shift money b/t D and P

only if good consequences in future.

2) NOMINAL DAMAGES: sometimes granted if P can prove the elements of a tort but has suffered no actual damages.

3) PUNITIVE DAMAGESa) Not available to all Ps, only to those who can demonstrate that they were victims of certain “aggravated” forms of

mistreatment involving “malice, insult, oppression, or wanton or willful violence.”i) Punitive damages justified only when evidence indicates that D acted wantonly in causing the injury or with

such conscious indifference to consequences that malice may be inferredii) P’s burden of proof: P must present clear and convincing evidence that D acted wantonly, willfully, or

recklessly towards P.(1) Clear and convincing standard is more onerous than preponderance standard, and less demanding than

beyond a reasonable doubt standard.b) Rule: P is ineligible to receive punitive damages if she can establish only that D wronged her by acting carelessly

toward herc) Rule: Negligence can support punitive award if P establishes that D’s unreasonable conduct demonstrates reckless

indifference to rights of others - wanton disregard/indifferencei) Reckless indifference: acts in question that pose a risk of harm that is substantially greater than that which is

necessary to make his conduct careless.(1) 2 types of reckless indifference:

(a) Wanton disregard : reason to know of facts which would lead a reasonable man to realize that those dangers attend his conduct(i) Permits factfinder to conclude that actor acted recklessly even though he wasn’t aware of dangers

posed by his conduct at the time of acting(b) Deliberate indifference : conscious choice of a course of action with knowledge of serious dangers to

others involved(c) Distinction b/t the two is degree to which actor is cognizant of risks posed by this conduct

d) Jury awards of punitive damages are rare – tend to be given for intentional torts.

Amanda Lee, Hershovitz – Torts, Fall 2009 28

Page 29: Torts outline

e) Bifurcation: to reduce risk that evidence of D’s wealth will shade juror’s judgments, bifurcate proceedings.i) Evidence of D’s wealth is relevant to jury’s determination of size of punitive award – need to factor in b/c D’s

wealth may have factored into decision to behave as he did, and larger award may be necessary to send a meaningful message to wealthy individual

ii) For compensatory damages, juries are shielded from D’s wealthf) Statutory “safe harbors”: instances where P who might otherwise be eligible to receive punitive damages will be

blocked by statutes that bar such an award in particular contexts p.499g) Criticism that punitive damages are crossing boundaries b/t crime and torts, public and private law, and

punishment and compensationh) Cases

i) National By-Products, Inc. v. Searcy House Moving Co .: NBP driver crashed into a car, causing it to hit a truck carrying a house that was stopped in the road – the 2 ppl in car were killed. NBP also hit the house and crashed into another car. House damaged. Jury gave moving company (house) $100k in punitive damages.(1) Holding: Court reversed punitive damages award - Facts don’t show that D intentionally acted in such a way

that the natural and probably consequence was to damage P’s property. Facts don’t show that D knew that act of negligence was about to cause damage and still cont’d to cause that damage.(a) Driver either didn’t apply brakes, or he applied them but they didn’t function properly.

(2) DISSENT: no longer require malice for punitive damages – enough if D acted recklessly, wantonly, or with a conscious indifference to safety and welfare of others(a) NBP driver was driving at excessive speed – driving overloaded truck with defective brakes on a

highway and speeding – collision is a natural and probably consequenceii) Mathias v. Accor Economy Lodging, Inc. : Ps were bitten by bedbugs while staying in D’s motel.

(1) Holding: D’s actions were reckless b/c they knew of bedbugs and continued to rent out rooms that were known to be infested – unjustifiable failure to avoid a known risk. Deliberate indifference.

(2) Need punitive damages because D’s behavior was outrageous, but compensable harm was slight and difficult to quantify because large element was emotional.(a) If punitive aware is reduced, P might not have enough to finance lawsuit.

i) Purpose of punitive damages?i) Civil alternative to criminal prosecution of minor crimes

(1) Damages should be proportional to the wrongfulness of D’s actions(a) Modified when probability of detection is very low or crime is potentially lucrative

(2) Need to be reasonably clear standards for determining amount of punitive damages for particular wrongs, so that D has reasonable notice of the sanction for unlawful acts and can make rational determination of how to act

(3) Sanctions should be based on the wrong done rather than on status of D; person punished for what he does, not who he is (Aristotelian notion of corrective justice)

ii) Posner: Compensatory damages won’t always work - punitive damages assist in deterring and punishing criminal conduct that would otherwise escape sanction, b/c goes it undetected, or b/c victim won’t have sufficient incentive to bring suit in light of psychic or economic costs of litigation(1) Compensatory damages are difficult to determine in cases of acts that inflict largely dignatory harms –

injury too slight to give P incentive to sue, so might respond with violence(a) To limit P to compensatory damages would enable D to commit offensive act with impunity provided

that he was willing to pay - danger that his act might incite violence(2) Limit D’s ability to profit from fraud by escaping detection and (private) prosecution

(a) If tortfeasor is caught only ½ the time he commits torts, then when he is caught he should be punished twice as heavily in order to make up for the times he gets away

(3) Provide deterrence for wealthy D - punitive damages for when D treats harm to P as simply a cost of doing business – treat compensatory damages as the price to be paid for securing the “right” to injure others

(4) P might have difficulty financing lawsuit – makes it worthwhile to sue(a) D’s wealth isn’t sufficient basis for awarding punitive damages, but can consider when it enables D to

mount an extremely aggressive defense against P’s suit, making litigating costly for P and difficult for P to find a lawyer willing to handle its case.

iii) Damages – what tort law is about(1) Calabresi: forward looking aim of efficiently deterring accidents

(a) Economic view, forward looking – doesn’t care about actions, just want to figure out who’s going to pay for it.

(b) Assign liability to cheapest cost avoider(2) Pre-Calabresi: Institution that’s trying to do justice between the wrongdoer and victim

(a) Corrective justice view – money doesn’t really accommodate pain and suffering

Amanda Lee, Hershovitz – Torts, Fall 2009 29

Page 30: Torts outline

4) VICARIOUS LIABILITY: doctrine by which one person/entity is responsible for tortuous acts of another who is acting on her or its behalfa) RESPONDEAT SUPERIOR: an employer is subject to liability for tortuous conduct committed by its employee

i) vs direct liability: where employers acted wrongfully by failing to screen or supervise its employees, and this wrongful conduct helped bring about P’s injuries – negligent hiring, negligent supervision

ii) When an employer is held liable under RS, doesn’t relieve employee. Instead, employer and employee held jointly and severally liable to P.(1) P usu seeks recovery from employer b/c more likely be able to pay(2) Employer can seek reimbursement from employee, but usually doesn’t because it has liability insurance

footing the billiii) To find RS, inquiry should be whether the risk was one that may be fairly regarded as typical of or broadly

incidental to the enterprise undertaken by the employer(1) Old test: close link required between acts of agent and profit accruing to employer

(a) Employer-benefit requirement met whenever broad potential effects on morale and customer relations exists, or where employers have implicitly permitted/endorses recreational practices that led to harm

(2) New test (CA): RS is concerned with allocation of the cost of industrial injury(a) Whether the employee’s conduct is not so unusual that it would seem unfair to include the loss

resulting from it among the costs of the employer’s business(b) Integrating principle: employer should be liable for those faults that may be fairly regarded as risks of

his business, whether they are committed in furthering it or notiv) Scope of employment – detours and frolics

(1) Employee detour: slight deviation from expected course or route that an employee would take in the course of doing his job

(2) Employee frolic: an employee who so far deviates from his employment obligations that he is deemed to be on his own business

(3) An employer is subject to liability for torts committed by an employee on a detour, but not on a frolic.v) Scope of employment – intentional torts and state and federal law

(1) Older “purpose” test: cts excluded intentional wrongful acts of employees as being outside scope of employment

(2) Modern “characteristic activities” test: cts have held that some but not all forms of intentional employee wrongdoing can be attributed to employers(a) Employers are liable when acts committed while serving the employer’s purposes—i.e. a bouncer

forcefully removing a patron at a nightclub.(b) Employers are not liable when the employee was pursuing an independent course of conduct not

intended by the employee to serve any purpose of the employer. (3) FTCA excludes such liability on federal employees unless intentional tort is committed by an investigative

or law enforcement officer.vi) Westfall Act – personal immunity for govt employees. Person injured by tortuous conduct of federal employee

acting w/in scope of employment can’t recover from employee, only from gov’t.vii) RS doesn’t apply to independent contractors

(1) Factors for determining whether person is employee or contractor (more on p.511, note 10 – exam advice – know how to differentiate b/t the two!)(a) Extent of control the principal has exercised in practice over the details of agent’s work(b) Whether type of work done by the agent is customarily done under a principal’s direction or w/o

supervision(c) Whether agent or principal supplies the instrumentalities required for the work and the place in which

to perform it.(d) Whether agent’s work is part of principal’s regular business(e) Whether principal and agent believe that they are creating an employment relationship

(2) Treating physicians at hospitals often are independent contractors – some just have permission to use hospital’s facilities as they are available

(3) Non-delegable duty doctrine: cts have sometimes prohibited commercial landowners from disavowing carelessness by independent contractors performing work on premises

viii)Ex: Taber v. Maine: D, an off-duty Navy serviceman, got drunk on the base and left base driving drunk, injuring P.(1) Holding that Maine’s negligent conduct was in the scope of his employment:

(a) Drinking on base during off-duty hours is a commonplace(i) Drinking by servicemembers can be viewed as imp’t for morale (benefit to employer – old test)

(b) Foreseeable that drunken servicemember would leave base and cause damage(c) Not unfair or unreasonable to impose cost on gov’t – military given pervasive control on its employees

on base – integrating principle – actions attributable to business, regarded a risks (new test)

Amanda Lee, Hershovitz – Torts, Fall 2009 30

Page 31: Torts outline

(2) Westfall Act - Maine didn’t get it b/c he was representing himself and failed to invoke the statutory provision.

(3) Difference b/t direct and vicarious liability:(a) Direct: Military is trying to control Maine, so gov’t is liable(b) Vicarious: Maine is liable so gov’t must pay.

ix) Ex: Rogers v. Kemper Construction: where employees drinking after hours got into fight – employer held liable(1) Ct held that where social/recreational pursuits on employer’s premises after hours are endorsed by

express/implied permission of employer and are conceivably of some benefit to employer, or if such activities have become a customary incident of employment relationship, and employee engaged in such pursuits after hours is still acting w/in scope of employment

x) Justification for RS: what an employer does in the world is what an employee doesb) Parent’s aren’t vicariously liable for careless acts of their children. May be directly liable.

i) Exception in case of parents letting children drive their cars. If child drives carelessly and injures someone, parents, as car owner, are held liable.

5) JOINT LIABILITY AND CONTRIBUTION:a) Available when:

i) When 2+ tortfeasors act concurrently or in concert to produce a single injury, they may be held jointly and severally liable.

ii) Indivisible harms: Where 2+ tortfeasors don’t act concurrently or in concert, but injuries because of their nature, are incapable of any reasonable or practicable division or allocation among multiple tortfeasors.

b) Doesn’t apply when 2+ tortfeasors don’t act in concert or concurrently, they’re not joint tortfeasors. Instead, their wrongs are independent and successive.i) Initial tortfeasor liable for entire damage resulting from his wrongful acts. Successive tortfeasor liable only for

separate injury his conduct caused.ii) Ex: Ravo v. Rogatnik: P was rendered severely retarded by the mistakes of two separate doctors before and

during/immediately after her birth.(1) Holding: In cases in which the injury is not divisible, tortfeasors who neither act in concert nor concurrently

can be held jointly and severally liable, even though the fault is split 80%-20%, both parties are on the hook for 100%.(a) Being 20% responsible for damages is different from being 20% responsible for injury

(i) How negligent you are doesn’t necessarily translate to how much harm you did p. 518-519c) Joint tortfeasors are each liable for the full amount of P’s damages, and may be sued for those damages either singly

or along with the other tortfeasors.i) Rationale:

(1) Places the duty on Ds to collect from each other (contribution/restitution) so that the plaintiff gets all of the damages she earned regardless of whether one D is insolvent or not.

d) Unknown defendants :i) Some states permit juries to assign fault to non-parties and others do not.

(1) Ex: Bencivenga v. J.J.A.M.M., Inc.: P was attacked on the dance floor of a club by an unknown person, and sued the bouncers and the owner of the club for negligent or intentional conduct. D wanted fault apportioned to unknown person as well.(a) Rule: Damages cannot be apportioned to an unknown intentional tortfeasor in NJ, and the burden of

naming the unknown party is on D. D failed to name him, so D can’t have damages reduced in proportion of unknown tortfeasor’s fault.(i) Policy reasons: amount of P’s judgment and amount of D’s liability vary on whether fictitious

person is considered. D has incentive to name additional tortfeasors; in better position to identify other Ds. In NJ, only held jointly and severally liable if above 60%.

(b) Club also had the duty - proprietor of premises to which public invited for business purposes, owes a duty to exercise reasonable care to discover intentionally harmful acts of third parties are being done or likely to be done, or give warning adequate to enable patrons to avoid the harm, or otherwise to protect them against the harm

e) With comparative responsibility, does joint and several liability even make sense anymore? i) Comparative responsibility: Ds pay based on jury’s % of apportionment of fault vs. pro rata basis where

damages divided evenly among responsible partiesii) Implicitly, the adoption of comparative responsibility negates the idea of an indivisible injury because it forces a

jury to apportion fault.iii) Some courts have concluded that comparative responsibility is inconsistent with the doctrine of joint and

several liability for indivisible injuries – states have begun to eliminate or restrict joint and several liability

Amanda Lee, Hershovitz – Torts, Fall 2009 31

Page 32: Torts outline

f) Hershovitz: 5 possible damages rules, different allocations of risk of D insolvency:i) Traditional rule: for indivisible injury, even when jury apportions responsibility, Ds are jointly and severally

liable.(1) If D2 is unreachable or insolvent, D1 will have to bear full cost of the missing share of damages.(2) D1 can bring a claim against D2 – right of contribution among joint tortfeasors

(a) A joint tortfeasor is barred from recovering contribution from the other tortfeasor if the other tortfeasor has settled with P

ii) CO rule: each D won’t pay more than his allocated % - D only responsible according to % fault jury assigns.(1) If one D is insolvent, P will have to bear the cost

iii) If a D is unreachable or insolvent, allocate missing share among all the parties who have contributed to P’s injuries (including P himself sometimes)(1) Ex: If P, D1, D2 all at fault. P 10%, D1 50%, D2 40%. But D2 is insolvent, so divide the 40% according to the

fault of P and D1 – P gets 1/6 of it, D1 gets 5/6.iv) Split remaining part that insolvent D can’t pay 50-50v) NJ rule: joint and several liability possible where tortfeasors’ % of responsibility is above a certain threshold.

(1) If any D is over 60% liable, he can be held to pay the whole. (2) If under 60%, can only be held to his proportion.

PART 3: STRICT LIABILITY, PROPERTY TORTS, ULTRAHAZARDOUS ACTIVITIES1) STRICT LIABILITY (Liability w/o fault) – liability that attaches w/ proven or presumed carelessness, recklessness or

intentional wrongdoinga) Applies when:

i) Conduct that interferes with a person’s interest in the possession, use, or enjoyment of the land or personal possessions

ii) Ultrahazardous activities, esp use of explosives and keeping wild animalsiii) Manufacture, design, and sale of defective products by manu and other commercial actors

b) Even though have strict liability, P still needs to show actual and proximate causec) Ps prefer strict liability because they don’t have to prove unreasonable/reasonabled) When plead strict liability, you want to please negligence as well, and vice-versa

2) Property Torts – TRESPASSa) Definition: Requires one person to act so as to cause a certain kind of interference w/ property possessor’s rights of

exclusive possession and controli) Interfering act must be intentionally undertaken – though need not be any intention to do harm to P or to

invade property that actors knows to be owner, or need not be any unreasonable conduct. It’s enough if:(1) D intentionally invades land(2) And P owns or possess that land(3) Therefore, D is held liable for damages caused by trespass even if he didn’t know that he was trespassing

on P’s land.(4) Reasonableness doesn’t matter, that’s why it’s very similar to strict liability.

ii) Very minimal interferences are treated as actionableiii) Trespass can occur by means of inaction.

(1) Actor invited, but then consent withdrawn. Failure to leave can provide basis for trespass.(2) Failure to remove an object from another’s land is trespass, even if originally not tortuous.

iv) Trespass may be committed on, beneath, or above surface of the earth – usually related to cases involving underground leakage of gas or toxic waste

v) Trespass examples:(1) D intentionally enters P’s land.(2) D refuses to leave P’s land.(3) D intentionally causes another person or object to enter P’s land. (4) D fails to remove an object from P’s land that D is obligated to remove.

vi) No requirement that invasion in question damage or render less valuable P’s property. Absence of such harm will affect the damages that P gets.

vii) Kopka rule: special subset of physical injury claimants can recover even w/o showing carelessness of D, only if claimant can establish that injury was product of trespass

Amanda Lee, Hershovitz – Torts, Fall 2009 32

Page 33: Torts outline

viii)Other remedies available for trespass:(a) Obtain ct order enjoining ongoing trespass activity(b) Injunction ordering removal of structure(c) Possessors of land may sometimes employ self-help to defend their property rts, but only insofar as

such efforts are reasonableix) Ex: Burns Philp Food, Inc. v. Cavalea Contl. Freight, Inc.: BPF built a fence on CCF’s land, mistakenly believing

that the land belonged to them. BPF paid taxes on the land for several years, and now filing suit for restitution. CCF counterclaimed by claiming that building the fence was a trespass.(1) Court held that, even though BPF did not realize it was trespassing onto CCF’s land, BPF should be held

strictly liable for the trespass. (a) Trespass is entry w/o consent. BPF did not have CFF’s consent.

b) Trespass and Necessity defensei) Doctrine of Necessity: necessity, and an inability to control movement inaugurated in the proper exercise of a

strict right, will justify entries upon land and interferences with personal property that would otherwise have been trespass(1) Doctrine of necessity applies with special force to preservation of human life

ii) Public necessity: where private citizen is entitled to use or destroy another’s property in order to avert a greater harm to the public w/o suffering any sanction(1) No need to pay P for damages – public necessity is a complete privilege

iii) Private necessity – incomplete privilege to commit trespass(1) D does damage to P’s property because there is no less damaging way to protect D’s property.(2) D is still liable for compensatory damages that result from his exercise of the privilege.

iv) Cases(1) Ploof v. Putnam : D owned island with a boat dock that P wanted to use to save his family while they were in

a boat at storm(a) Ct held no trespass – preservation of human life.(b) But there was an incomplete privilege to commit trespass because while dock owner had to let boat

stay, boat owner has to pay damages.(c) The purpose of this doctrine is to give an at risk D, the right to damage P’s property to prevent great

harm to themselves or their property.(i) If a boat were unmoored during a storm by the dock owner to prevent destruction of the dock, the

boat owner could recover from the dock owner(2) Vincent v. Lake Erie Transp. Co .: A ship doesn’t undock after unloading cargo b/c of dangerous storm.

During storm, it has to repeatedly re-tie itself to the dock as ropes break. Ship damages dock. (a) Holding: An act of God can be a defense unless there are deliberate efforts to protect one’s own

property at the expense of another’s—in this case, D held ship deliberately in such a position that damage to dock results. Preserved ship at expense of dock – D must pay for damages the dock incurred from his actions. (i) Reasonable for D to be there, but trespass is strict liability.

(b) DISSENT: D exercised due care, damage to dock not his fault. Dock owners take the risk of damage to their docks.

(c) Did court reach the right result? Might not matter who’s responsible.(i) Some want to see that case as about restitution. Restitution is D’s gain – D benefited.

(d) Coase theorem: about world w/ no transaction costs(i) Legal rule you adopt doesn’t matter in world with no transaction costs, but real world has

transaction costs(ii) With no transaction costs where both parties know all information, they can reach a bargain

effortlessly1. Whether boat goes or stays doesn’t matter which legal rule you adopt

(iii) Bilateral monopoly: both parties stuck negotiating with each other and only each other – very high bargaining coasts1. Possible may not reach agreement

World A World BBoat $5000 $500

Dock $500 $5000

Amanda Lee, Hershovitz – Torts, Fall 2009 33

Page 34: Torts outline

(iv) World A 1. If rule that says boat must pay for damages, boat will stay – damage to dock cheaper.2. If dock is responsible, boat will still stay – give dock $500+ to stay – cheaper than leaving.

(v) World B 1. If loss on boat, boat’s going.2. If loss dock, boat’s going because too expensive to stay.

a. Maybe dock owner will pay boat to leave.(vi) Economists recognize that our world does have transaction costs, so they say in order to minimize

them, people could reach solution that they would reach in no transaction cost world1. Allocate risk to boat, reduce transaction cost by placing risk on party that has more information

(value of cargo, repairs of ship, etc ) so boat liable (what court said)a. This is an effort to minimize transaction costsb. If place loss on dock owner, dock will always have to have someone there – high

transaction costs. Boat owner may be the cheapest cost avoider.(vii) Ct’s view that boat was liable b/c they used dock for won benefit (corrective justice).

3) ULTRAHAZARDOUS ACTIVITIESa) Engaging in ultrahazardous activities (e.g. keeping wild animals or explosives) subjects a person to strict liability –

no negligence is required if someone is injured in the course of the ultrahazardous activity.i) Sec 519 Restatement – any party carrying on abnormally dangerous activity is strictly liable for ensuing

damagesb) Rule of law: person who, for his own purposes, bring on his land and collects and keeps there anything likely to do

mischief if it escapes, must keep it in at at his peril; and if he doesn’t do so, is prima facie answerable for all the damage which is the natural consequence of its escapei) If it does escape and cause damages, D is responsible, no matter how careful he may have been and whatever

precautions he took.ii) Ques is not whether D has acted with due care and caution, but whether his acts have occasioned the damage.

(1) D will be liable when he damages another y a thing or activity unduly dangerous and inappropriate to the place where it is maintained, in light of the character of that place and its surroundings

iii) Ex: Rylands v. Fletche r : D built a reservoir on his property, which leaked into vertical shafts and flooded P’s mines.(1) If D had used land for purpose that in ordinary course of enjoyment of land, and water had accumulated and

flood P’s mine – not liable. P could have put up barrier to protect himself.(2) However, if D, not stopping at the natural use of their land, had desired to use it for a non-natural use, and

as consequence water flooded P’s mine – D is liable.(a) Reservoir not natural – introducing risk. If using land in non-natural way, then responsible for the

damages it causes.(3) Rylands creates strict liability in the sense of relieving P of usual burden of proving fault, thereby subjective

D to liability unless it could prove lack of fault(4) Rylands strict rule is applied when knowledge of damage done, or threat to public is established

c) Defining abnormally dangerous activity: i) 6 factors to determine whether activity is abnormally dangerous

(1) Existence of high degree of risk of some harm to person, land, or chattels of others(2) Likelihood that the harm that results from it will be great(3) Inability to eliminate the risk by the exercise of reasonably care(4) Extent to which the activity is not a matter of common usage(5) Inappropriateness of the activity to the place where it is carried on(6) Extent to which its value to the community is outweighed by its dangerous attributes

ii) Any one of the factors isn’t necessarily sufficient, and ordinarily need several of them required for strict liability.iii) Not necessary that each be present, especially if others weigh heavilyiv) Essential ques: whether the risk created is so unusual, either b/c of its magnitude or b/c of circumstances

surrounding it, as to justify the imposition of strict liability for the harm that results from it, even though it’s carried on w/ reasonable care

v) 3rd restatement draft defines it abnormally dangerous activity as:(1) Creates a foreseeable and high significant risk of physical harm even when reasonable care is exercised by

all actors and,(2) Is not one of common usage

vi) Klein v. Pyrodyne Corp .: P was injured during a fireworks show by a firework that shot sideways – dispute as to why the sideways shot occurred.

Amanda Lee, Hershovitz – Torts, Fall 2009 34

Page 35: Torts outline

(1) Ct uses 6 restatement factors to impose strict liability - setting off fireworks displays is an abnormally dangerous activity because the first four of the six factors listed above are present.

(2) Where the risk is asymmetrical then we impose strict liability. d) Difference b/t reciprocal and non-reciprocal risks

i) Reciprocal risks: persons are exposing each other to roughly similar risks of injury(1) Fault ought to be standard of liability(2) Only exceptional risks that should generate tort liability in reciprocal risk situations

ii) Non-reciprocal risk: Risks are generated by unconventional activities; risks are unilaterally imposed by one actor upon others(1) Fairness requires rule of strict liability, otherwise unilateral risk-create will enjoy rt to impose risks on

others even though those others won’t have the same righte) Keeton: 2 types of fault

i) Fault: involves activity that violates norms of behavior even granted that the actor who engages in the behavior has made provision for payment of any damages his behavior causes(1) Ex: careless driving

ii) Conditional fault: risky conduct that we are nonetheless prepared to treat as permissible if the actor who undertakes it stands ready to provide reasonable compensation to those injured by such conduct(1) liability w/o fault

f) Rationales for imposing strict liability for dangerous activities:i) Leads to less high-risk activity and fewer accident losses from it.ii) Encourages actors who engage in dangerous activities to reduce the cost of accidents by taking extra

precautions.iii) Losses should be placed on the party who can most easily spread the costs of the activity, which is usually the

company or individual engaging in the dangerous activity.

PART 4: PRODUCTS LIABILITYFROM NO-DUTY TO STRICT LIABILITY1) P can recover for a harm caused by a defect in a product through the body of law called strict products liability. NOTE:

PRODUCTS LIABILTY IS NOT STRICT LIABILITY.a) Products liability eliminated the duty and breach elements of a negligence claim.b) The seller of a defective product is subject to liability to the consumer even if he has exercised all possible care in the

preparation and sale of the product2) Escola v. Coca Cola Bottling Co.: P, an employee at a restaurant, was injured while stocking the cooler at her workplace

when a Coca Cola bottle exploded in her hand.a) Holding: The majority applied the doctrine of res ipsa loquitur to find D liable.

i) But, res ipsa doesn’t really seem to work in this case as the employee and the trucker had the bottle in their control so it is not clear that D must be responsible for the negligence – fails on exclusive control, majority just stretches.

ii) Traynor’s concurrence moves product injury into strict liability – no point in pretending it’s negligence, reality is that the holding is liability w/o negligence:(1) Should recognize that manufacturer incurs absolute liability when places article on market knowing that it’s

to be used w/o inspection, and proves to have a defect that causes injury(2) Even if no negligence, public policy demands responsibility be fixed wherever it will most effectively reduce

hazards to life and health manufacturer can anticipate some hazards and guard against recurrence, public can’t(a) Manufacturer will get insurance cost will be built into insurance. Consumers pay very little costs

since spread over all of them to pay for that one customer that gets the broken bottle(3) Even though intermediaries, manufacturer’s obligation to consumer is still there.

iii) Couldn’t bring warranty claim because:(1) P wasn’t the person who purchased the bottle(2) She was not a user of the product(3) Criminal statute for foodstuffs – she doesn’t consume product, she was just handling the bottle.

3) Traynor stated rule twice, slightly differently – a) First way: A manufacturer is strictly liable in tort when an article he places on the market, knowing that it is to be

used without further inspection for defects, proves to have a defect that causes injury to a human being.b) Second way: It is sufficient that P proved that he was inured while using product in the way it was intended to be

used, as a result of a defect in design and manufacture of which P was not aware, which made the product unsafe for its intended use.i) Second standard requires intended use of the product, and is thus more restrictive.

Amanda Lee, Hershovitz – Torts, Fall 2009 35

Page 36: Torts outline

4) Traynor’s justifications for new conception of liability:a) Obligation-based rationale: Manu owe consumers a demanding obligation to be vigilantb) Deterrence rationale: Manu best suited to take precautions; should be given strong incentives to do soc) Compensation-insurance rationale: Manu best situated to spread costs of accidental injuries caused by their

productsd) Causation-strict liability rationale: Responsibility for injury stems from having marketed a product that cause injury,

regardless of negligencee) Compensation-equality rationale: Victim’s entitlement to compensation shouldn’t depend on nature of conduct that

cause itf) Litigation-structure rationale: Analysis of disparities in power in litigation concerning evidence and procedureg) Judicial-candor rationale: Assertion that if 2 ways of structuring law lead to same result, more open and direct

structure preferable5) Other arguments for strict liability over negligence standard for products liability:

a) It’s cheaper for P’s and courts b) Allows manufacturers to anticipate the harm using cost spreading rationales, thus making the manufacturer the

insurer, but seems more expensive than typical insurance. c) Even if both parties are innocent, it’s better to place responsibility with the causer.d) Strict liability makes its easier for P to win, thus D is more likely to take precautions

6) Will strict liability standard affect manufacturer behavior? a) Posner argues that it doesn’t matter whether its strict liability or negligence because either way companies will decide

whether paying out damages or fixing the problem is cheaperi) This will not change the precautions that D will employ - they will still do the more efficient thing for business

(B<PL).(1) However, they may produce less.

ii) Posner – strict liability doesn’t increase precautions, just decreases how many people enter/stay in the business – b/c although it doesn’t change what precautions are taken (still only take economically efficient precautions), does change how willing you are to enter into the business.(1) More nuanced/slightly anti-Posner view: there’s never perfect information, and everyone with a potential

claim doesn’t always sue. A strict liability standard makes it more likely that people will sue, so although that’s not taken into account with traditional B<PL analysis, manufacturers may take more precautions if, in light of potential that more people will bother to sue, it becomes cost-effective to do so.(a) Strict liability may change precautions taken that negligence system, because of imperfect information,

should take care of but doesn’t.

PRODUCT DEFECTS1) Greenman v. Yuba Power Prods., Inc .: recognized as the first real strict products liability case; P was using a combo

power tool manufactured by D when the piece of wood he was working on flew out of the machine and struck P in the head.a) Holding: D should be held strictly liable for the injuries caused by its defective product.

i) To impose strict liability, not necessary for P to establish express warranty – not appropriate for courts to adopt in actions by injured consumers against manufacturers with whom they have not dealt with(1) Recognized that liability not assumed by agreement, but imposed by law. Manufacturers can’t define scope

of its own responsibility for defective products.ii) Can establish liability based on presence of defect – manufacturer is strictly liable when an article he places on

the market, knowing it is to be used w/o inspection of defects, proves to have a defect that causes injury b) P must prove that he was injured while using the product in the way it was intended to be used as the result

of a defect in design and manufacture of which P was not aware, which made the product unsafe for its intended use.

2) After Greenman, Restatement (Second) of Torts § 402A: Special Liability of Seller of Product for Physical Harm to User or Consumera) “Defective condition”: one not contemplated by ultimate consumer which will be unreasonably dangerous to himb) “Unreasonably dangerous”: Article sold must be dangerous to an extent beyond that which would be contemplated

by the ordinary consumer who purchases it, with ordinary knowledge common to the community as to its characteristics

Amanda Lee, Hershovitz – Torts, Fall 2009 36

Page 37: Torts outline

i) Purpose of this qualification was to prevent possibility that manufacturer of a product with inherent possibilities for harm would become automatically responsible (ex: butter, drugs, whiskey and cars).(1) consumer expectations test

c) “Defective condition unreasonably dangerous” requires only single finding – don’t need to split it upi) Problem with splitting it up – advantageous to characterize defect either in design or manufactureii) Requiring both would put greater burden on P than articulated in Greenman

d) § 402A in practicei) Most states adopt 402A without the unreasonably dangerous part.

(1) “Unreasonably dangerous” modifies the product, not the manufacturer’s conduct, but many courts still worry that it reintroduces negligence.

ii) In most states, P’s negligence is compared with the strict liability of the manufacturer.iii) Unreasonable uses don’t matter as long as the use was an intended use.

3) Third Restatement adds Sec 2(b): must render product “not reasonably safe”i) Better than “unreasonably dangerous requirement”?

b) Most cts don’ts expressly include such a phrase in their formulations of standard of liability for products cases4) Cronin v. J.B.E. Olson Group : P, an employee for a bakery, got into a car accident. The impact broke the hasp meant to

keep the bread trays in place, and P was hit by the trays as they fell.a) Issue: Whether injured P seeking recovery upon theory of strict liability must establish not only that product

contained a defect which proximately caused his injuries, but also that such defective condition made the product unreasonably dangerous to the user or consumer.

b) Court held that “unreasonably dangerous” introduces an element of ordinary negligence back into products liability claims, which places undue burden on the plaintiff – consumers aren’t in position to say what defect wasi) Using the unreasonably dangerous standard presents the danger that the jury will decide that manufacturers

are shielded from liability as long as the product did not fall below the ordinary consumer’s expectations as to the product’s safety—this is negligence, not strict liability.

c) Most jurisdictions do not accept the unreasonably dangerous requirement and just determine whether or not there was a defect in the product.

d) Is it ok to make elements so Ps can win cases? i) Strong emphasis on compensation rationale, risk rationale – thinking of it in terms of insurance.ii) At the time Cronin opinion written, wanted to spread defect cost among consumers.

5) BASICS OF A PRODUCT LIABILITY CLAIMa) Actor A is subject to liability to P in products liability if:

i) P has suffered an injury(1) Harm suffered is usually a physical harm(2) Owner of product can’t invoke products liability to recover for a defect that causes damage to product itself. (3) Can invoke of product damages property other than product itself

ii) A sold a product(1) What is a product?

(a) Services, real property, human body parts, live animals (depends on context), textual material (maps and charts sometimes exempt from this) aren’t products

(b) Intangibles resides at margin of product/non-product(c) Used products not subject to strict liability(d) Some jurisdictions deem something a product, but exempt it from product liability (ex: vaccines,

prescription drugs)iii) A is a commercial seller of such products

(1) Seller must be in business of selling or marketing product - manufacturers, retailers, distributors(a) Sellers and strict liability: often attaches liability to person that played no role in design, manufacture, or

inspection of product(b) Sellers and law of warranty: relation to contract law(c) Distribution chains and indemnification

(i) Indemnification clauses: permit one to sue another to recover costs from litigation and liability for injuries stemming from products supplied

(2) Item “sold” if actor took the steps to place the product on the market figured into the distributional chain through which product is placed on market

(3) If have service vs sale, if cts deem D engaged primarily in service, won’t treat D as seller of product(4) A lease counts as a sale; can still sue

Amanda Lee, Hershovitz – Torts, Fall 2009 37

Page 38: Torts outline

(5) Component part: manufacturer and seller of component part liable(a) If manufacturer of a component part can show that there’s nothing intrinsically wrong with his

component part (problem with larger product was that it didn’t function optimally with that component, but it wasn’t the component’s fault), can often avoid liability.

(b) Less likely to avoid liability, though, when there’s been a long relationship and component part manufacturer knows how his part is used.

iv) At the time it was sold by A, the product was in a defective condition; andv) The defect functioned as an actual and proximate cause of P’s injury

(1) Don’t have to buyer or user of product to sue. Could be bystander.vi) injury and causation elements are parallel to negligence; duty and breach element are absent

6) DEFECTSa) A cause of action for products liability exists only if product in question is defective, and only if defect that caused

the injury was present in the product when sold or marketed by the defendantb) If consumer buys product knowing its defective, seller isn’t strictly liable warning?c) MANUFACTURING DEFECT: if diverges from manufacturer’s own specifications for product; can be a single product

i) Easiest kind of defect to proveii) The defect must emerge while the product is in the control or possession of the manufacturer to be charged to

the manufacturer.iii) Most jurisdictions allow P to use circumstantial evidence to prove the existence of a manufacturing defect

because it is often hard for P to prove that such a defect exists.iv) Gower v. Savage Arms Inc. : P bought a gun from D and shot himself in the foot while hunting because the gun

appeared to still be in the “safety” position. He made four distinct claims(1) Ct held there was a manufacturing defect because one of a same model wouldn’t have had the same defect(2) Hypotheticals:

(a) If borrowed gun from friend, can’t sue b/c friend isn’t a “seller.” If bought gun from friend, can’t sue b/c friend isn’t a “seller.”(i) Could sue friend for negligence if bought or borrowed, though.

d) DESIGN DEFECT: there is flaw in plan or specifications for product; defects inherent in entire line of productsi) Factors that tend to be considered in deciding whether there’s a design defect:

(1) The significance of the risk of physical injury posed by the particular design (2) How ordinary consumers would expect the product to function(3) Whether there is a feasible, safer, and affordable design.

ii) Two tests (no longer willing to leave it up to jury like in Cronin of whether there’s a defect):(1) Consumer expectations test: product is defective in design if aspects of its design render it more

dangerous than an ordinary consumer would expect it to be(a) Advantages

(i) Intuitive—allows a jury to make a common-sense evaluation of the product(ii) Does not require a plaintiff to prove a safer way to design the product

(b) Concept of a flaw or a defect can be given context by comparing the actual product to a prototype in the mind of the ordinary customer

(c) Generally, can’t use consumer expectations test where expert testimony is necessary. Expert testimony will be allowed, though, when consumer is specialized consumer with special expectations.

(2) Risk utility test: Product is defective if risks of its design outweigh its utility – RISK > UTILITY?(a) Derived from negligence(b) Should the product be designed differently given the relative risks and benefits posed by its current

design?(c) Foresight test: knowledge of risks at time product was made(d) Hindsight test: whether manufacturer is liable based on knowledge of product’s risks at time of trial –

would they have put on that product had they know about the defect?(e) Most jurisdictions apply a foresight test nowadays, instructing juries to base their decisions on risks

that were foreseeable to the manufacturer.(f) From Dean Wade

(i) Ask whether product so likely to be harmful to persons that a reasonable prudent manufacturer, who had actual knowledge of its harmful character would not place it on the market?

(ii) Ask whether magnitude of risk created by the dangerous condition of the product was outweighed by the social utility attained by putting it out in this fashion

Amanda Lee, Hershovitz – Torts, Fall 2009 38

Page 39: Torts outline

(iii) 7 Factors for whether or not liability to bring design defect case to court p.8901. Usefulness and desirability of product – its utility2. Safety aspects – likelihood that it’ll cause injury and probably seriousness of injury3. Substitute product?

a. Controversial in 3rd restatement that availability of alternative design should be a necessary element of prima facie case

4. Manu’s ability to eliminate unsafe character of produce w/o impairing its usefulness or making it too expensive to maintain its utility

5. User’s ability to avoid danger6. User’s anticipated awareness of dangers inherent in the product and their avoidability7. Feasibility of manufacturer of spreading loss 8. Factors 1 and 2 seem to be Hand formula9. Factors 5 and 6, idea that problem creating harm shouldn’t qualify as defect if means by

which risk could be avoided and seller has ensured that consumers are aware of risk and how to minimize it.

(iv) Not necessary to find that D had knowledge of harmful character of product in order to determine that it was not duly safe

(3) Today’s controversy of which test to use – consumer expectation or risk utility?iii) Cepeda v. Cumberland Eng’g Co .: P was operating a pelletizing machine without the guard on when his hand got

caught inside the machine and four fingers were lost.(1) Holding: The consumer expectations test is not appropriate in cases involving industrial machines because

a consumer does not have expectations about how an industrial machine should work.(a) The court instead balances the costs and benefits of the machine’s design using the risk-utility test, and

holds that the machine should have been designed so as not to operate without its guard.(i) By using risk utility instead of consumer expectation the court circumvents P’s possible

knowledge of the likelihood of the injury. (ii) If you use the risk-utility test, are you basically just analyzing under negligence standards? NO.

1. Asks a jury to look at the manufacturer’s decision in hindsight, with all the knowledge they have now, rather than the knowledge they had at the time the decision was made.a. Most states have switched to foresight test now.

2. This is what differentiates the risk-utility test from negligence – negligence focuses on whether it was reasonable at the time decision was made, w/ what they knew then.

(iii) Under risk utility test, manufacturer needs to anticipate misuse. If anticipated, then misuse doesn’t count as a comparative fault.

iv) Rationale for affirmative case of strict liability for design defects includes condition that the product isn’t being misused or abnormally used except where foreseeable.(1) Abnormal use isn’t an affirmative defense

v) Intended vs. Foreseeable Use: Product “misuse” and defectiveness(1) If P able to demonstrate a feasible design that would minimize injury generated in part by ordinary

consumer behavior, D held responsible to adopt that design(2) D’s view – has fulfilled its obligation to consumers by producing a product that is safe when used as

intended by seller(3) P’s careless use of product will not negate claim of defect unless carelessness is so gross to count as

unforeseeable to a reasonable seller at time of manufacture or sale.vi) Affirmative defenses

(1) With shift to comparative responsibility, cts have revisited and most still reject reasons against recognition of P’s fault as partial or complete bar to recovery

(2) Most jurisdictions, if there’s evidence of casually relevant unreasonableness on part of P, jury asked to assign % of responsibility to seller and P

7) FAILURE TO WARN OR INSTRUCT: Defective for lack of adequate warnings when safety requires that the product be sold with a warning, but the product is sold w/o a warning (or w/o an adequate warning); defect is omission of languagea) Whether a warning is required depends on whether the omission of the instructions or warnings renders the

product not reasonably safe.b) Henderson and Twerski: letting P recover for failure to warn increase in warnings proliferation of warnings

creates culture where consumers become dismissive of warnings

Amanda Lee, Hershovitz – Torts, Fall 2009 39

Page 40: Torts outline

c) Warnings must be adequate to notify consumer of the existence and nature of the hazard at issued) Eliminate knowledge component?

i) CA, Majority view that knowledge, actual or constructive, is requisite for strict liability for failure to warnii) Minority view: knowability requirement and admission of state of art evidence, improperly infuse negligence

concepts into strict liability cases by directing trier of fact’s attention to conduct of manufacturer/distributor vs condition of product

e) Difference b/t failure to warn in strict liability and failure to warn in negligence contexti) Negligence law: require P to prove that manufactuer/distributor didn’t warn P of particular risk for reasons

which fell below acceptable standard of case – what a reasonably prudent manu would’ve known and warned about

ii) Strict liability: require P to prove that risk made product unsafe and that D didn’t adequately warn of a particular risk that was known or knowable in light of the generally recognized and prevailing best scientific and medical knowledge available at time of manufacture and distribution; reasonableness is immaterial(1) Strict liability for failure to warn requires proof that D failed adequately to warn of a risk not just when risk

was known/knowable, but also when failure to warn of that risk renders product unsafe to its usersf) Black letter law: failure to warn of obvious danger is not actionableg) Anderson v. Owens-Corning Fiberglass Corp .: P contracted asbestosis through exposure to asbestos while working

with materials manufactured by D.i) Holding: Manufacturers are not required to warn about risks that were not known or knowable at the

time of manufacture. (1) Knowability is relevant to imposition of liability under failure to warn theory of strict liability.

(a) Compensation insurance rationale – spread losses among all users of product. Company acts as insurer; costs built into product.(i) Not going to insure against costs that you don’t or shouldn’t know about.

ii) D can introduce state-of-the-art evidence to prove that, at the time the product was manufactured, the manufacturer did not know about the risk.(1) State-of-the-art evidence looks toward what a reasonable manufacturer would have done at the time the

product was manufactured; this introduces an element of negligence into strict products liability.iii) Under Anderson, strict liability for failure to warn requires proof that the failure to warn of the known risk

rendered the product unsafe to consumers.(1) Rationale - how can D be required to enforce against unforeseeable risks?

iv) Requirement for P to bring state of the art evidence increases the costs for P to find experts and such. h) Who to warn?

i) Usually manufacturers’ duty to warn runs directly to consumers of productii) Sophisticated users principle: amount of information a manufacturer must provide with its product depends

on sophistication and background knowledge of relevant pool of consumers(1) Manu’s duty will be narrower b/c certain risks that wouldn’t be obvious to layperson, would be obvious to

sophisticated useriii) Learned intermediary doctrine: Warning is given not directly to the consumer, but to an intermediary; for

instance, in the case of prescription drugs, the warnings and instructions are provided to doctors.(1) Medical expert in best position to evaluate complex info provided by manufacturer concerning risks and

benefits to make an individualize medical judgment, based on patient’s particular needs and susceptibilities, as to whether patient should use product

(2) Exceptions: argument that with advertising directly to consumer, rationales for doctrine gone(3) Motus v. Pfizer, Inc. : P’s husband committed suicide after his doctor prescribed Zoloft to treat his depression.

P claims that D failed to warn the doctor about the risks of Zoloft.(a) P must prove not only that no warning was provided or warning was inadequate, but also that

inadequacy or absence of warning caused injury(b) D concedes its warning was inadequate. But P failed to show that inclusion of warning would’ve altered

Dr’s decision to prescribe Zoloft to Motus.(i) Heeding presumption: once P establishes that manu provided inadequate warnings, burden shifts

to D to show that adequate warnings wouldn’t have affected the doc’s conduct in prescribing the drug; burden shifting device that makes it easier for P to prove causation1. Idea that if warning, would be heeded2. Ct does not apply the presumption in this case

Amanda Lee, Hershovitz – Torts, Fall 2009 40

Page 41: Torts outline

(c) P couldn’t show but-for causation (Strict liability just gets rid of breach prong, not causation) (i) Failure to warn actions require P to prove that defect actually caused the injury

1. Raises question of whether someone would have selected a different course of conduct if different, or greater, info or warnings had been provided

(ii) P must prove that if D had given warning, would’ve read and followed.iv) Cost spreading: cases where D isn’t wrong, but well positioned to bear and spread cost of a hapless victim’s

injuries(1) Cts purport to reject loss spreading as itself sufficient to justify imposition of liability – product liability not

absolute, and manufacturer not insurer of consumers(2)

PRODUCTS LIABILITY DEFENSES1) Comparative negligence

a) Most jurisdictions treat comparative responsibility as they do in negligence cases—reduces P’s recovery by percentage of fault that the jury assigns to P.

b) A few states still adhere to Comment n of Section 402A of the Second Restatement, which held that P’s negligence is irrelevant in strict products liability.

2) Assumption of Risk a) Second Restatement: P’s deliberate assumption of the risk should bar her from recovery entirely.b) Today, many states now treat P’s conscious choice to encounter the risk posed by a defective product as a form of

comparative fault—reduces recovery rather than barring it altogether.c) All forms of P’s failure to conform to applicable standards of care are to be considered for the purpose of

apportioning responsibility between P and the product seller or manufacturer—assumption of the risk has, in effect, folded into comparative responsibility.

3) Misuse a) Two types of misuse

i) When P uses the product in such an unforeseeable way that the manufacturer could not reasonably have foreseen this type of use.(1) Courts will generally find that the product was not defective, even though it was dangerous when P used it

the way he did, because the product was never designed to be used like it was.(2) This type of misuse prevents P from establishing a prima facie strict products liability case because P cannot

show that the product was defective because he cannot show that it was used for a purpose the manufacturer intended or should have foreseen.

(3) P cannot recover at all in this case.ii) P misuses the product, but in a foreseeable way.

(1) If the misuse is foreseeable, manufacturer has a duty to warn against that use.(2) In this case, manufacturer pleads misuse as an affirmative defense.(3) This misuse will be treated as a form of comparative fault in a majority of jurisdictions.

b) Sometimes, misuse is conscious assumption of the risk—the deliberate choice to encounter a risk created by the defective product.i) Example: when P deliberately decides to use a machine while the guard is not on.ii) These cases are treated like assumption of risk, above.

4) Open and obvious danger (so no duty to warn)a) Defendants raise the open and obvious danger defense.b) Two types of open and obvious danger

i) Ones that can be designed out of a product(1) Example: a snowblower with exposed blades. (2) The danger is open and obvious, but the manufacturer could have easily added a shield over the blades.(3) This design fails both consumer expectations and risk utility tests.(4) Therefore, manufacturer cannot avoid liability by arguing that that the danger was open and obvious. If this

worked, manufacturers would not have a duty to design reasonably safe products.ii) Ones that cannot be designed out of a product

(1) The open and obvious danger is not the result of a design flaw.(2) Example: a knife. A knife cannot do its intended job unless it is sharp.(3) The risk cannot be eliminated without undermining the utility of the product.(4) Therefore, the design of the knife is not defective, despite its risks.(5) P can argue that, if a product has to be somewhat dangerous to do its intended job, the manufacturer could

have at least warned P of the risk posed by the product’s design. P switches to a failure to warn theory:(a) In failure to warn claims, the obviousness of the danger plays a different role.(b) P will lose because a manufacturer has no duty to warn people of dangers that are open and obvious.

Amanda Lee, Hershovitz – Torts, Fall 2009 41

Page 42: Torts outline

5) State of the art defense a) Is a product defective if it posed dangers that were unknown at the time of sale and could not reasonable have been

discovered through investigation and testing?b) More recent cases have concluded that manufacturers are not liable for failing to warn of a danger unless they knew

or should have known of the need for a warning. c) Third Restatement upholds this view. d) Is state of the art evidence a defense or is it an element of the prima facie case that is missing?

i) Some states treat it as a defense.ii) Others treat it as an element of the prima facie case—P cannot prove that the product was defective if he cannot

prove that the manufacturer knew or should have known about the danger before it was sold.

PART 5: INTENTIONAL TORTSCOMMON LAW DOCTRINE, BATTERY AND ASSAULT1) BATTERY: purposeful touching – intentional invasion of P’s person space; requires actual touching, either direct or

indirect, by the wrongdoera) Infliction of a harmful or offensive contact by an actor upon another with the intent to cause such contactb) Elements of battery: A is subject to liability to P for batter if:

i) A actsii) Intending to cause a contact with P

(1) Intent: substantial certainty that contact will occuriii) of a type that is harmful or offensive (of what a reasonable person would perceive)

(1) A tortfeasor has no defense if he did not mean to offend, or if he did not realize that his act would offend.(2) The question of offensiveness will often be for the factfinder.

iv) A’s act causes such a contact(1) Doctrine of Extended Personality: battery can be established by contact with anything so connected with the

body as to be customarily regarded as part of the other’s person and therefore as partaking of its inviolability

(2) Defendant need not be present at the time of the touching—setting a wire across a sidewalk and leaving, knowing that P will trip over it later.

(3) Types of contact: (a) Harmful contact: causes bodily harms such as bruising or broken bones(b) Offensive contact: contact violates prevailing social standards of acceptable touching (objective

standard – reasonable standard)(i) Same act of touching might be inoffensive in some context, yet offensive in others, depending on

who is doing the touching, who is being touched, what the relationship is between them, the setting in which touching occurs, etc.

(ii) However, if D touches P in a particular way, knowing that P is unusually averse to being touched in that way, D’s contact can be considered offensive even if it might otherwise be deemed inoffensive.

(iii) Ex: Alcorn v. Mitchell: When trial adjoined, D spit in P’s face.1. Spitting in someone’s face is considered offensive contact.2. Since no real damages occurred, court awarded punitive/vindictive damages

a. Damages were probably specific deterrence rather than general deterrencec) Ex: Leichtman v. Wlw Jacor Communications, Inc: blowing cigarette smoke into the face of an anti-smoking

advocate constitutes offensive touching. i) No matter how trivial incident, battery is actionable.ii) Smoker’s battery: court rejects holding a smoker liable for smoking in the presence of non-smokers in general

(1) Doesn’t want people to put themselves in “glass cages” – want to avoid a system where someone can say, “I’m sensitive, don’t touch me ever, it’ll be a battery” – can’t construct a glass cage around yourself and have every contact be battery. The plaintiff must be reasonably offended.

iii) P’s claim for battery can stand, although ct thinks it’s better to have an alternative dispute resolution outside of ct system to provide for disputes in a nominal case.

d) Vosburg v. Putney : 12 yr old D kicked 14 yr old P who sustained injuryi) Circumstances surrounding the contact can determine whether or not it was a battery - kid kicked another kid

in a classroom, not on the playground or during a physical “rough-housing” activity”. ii) The kid got a very bad injury because of a previous kick to his knee.

(1) Egg shell skull rule applied: doesn’t matter if Putney intended the injury to be as severe as it was, take plaintiff as you find him

(2) If defendant touches P, intending that contact to be harmful or offensive, D is liable for all injuries resulting from the contact, even if the injuries are beyond what was intended or expected by D.

Amanda Lee, Hershovitz – Torts, Fall 2009 42

Page 43: Torts outline

(3) A person suing for battery or assault does not need to establish that D intended the particular injury, only that D intended to make the harmful or offensive contact.

e) Negligence or Battery? When battery, D is on the hook for more damages, possibly punitive.

2) ASSAULT: when person intentionally acts so as to cause another to apprehend that he is about to suffer a certain kind of kind – imminent harma) Whether there’s an assault, depends more upon apprehensions created in the mind of the person assaulted than

upon undisclosed intentions of person committing the assaultb) Elements of Assault: A is subject to liability to P for assault if:

i) A acts,ii) Intending to cause in P the apprehension of an imminent harmful or offensive contact with P, or intending to

contact P(1) Intent example: one who attempts to batter P but misses is liable for assault if P is placed in apprehension of

a blow.(2) Many times when have assault, actor never intended to contact P – just wanted apprehension(3) Transferred intent

(a) If A intends to hit B, but hits C instead. A can’t use defense saying that she didn’t intend to hit C. Intent to hit B transfers to C.

iii) A’s act causes P reasonably to apprehend such contact(1) Apprehension

(a) Apprehension: the perception or anticipation of a blow, rather than fright.(i) P does not have to fear harmful or offensive touching; it is enough when he was aware that such

contact might occur. (b) Apprehension of imminent harm must be reasonable - would D’s act place a reasonable person in

apprehension of an unwanted contact?(c) Must actually AND reasonably apprehend harm to you.

(i) Person standing behind you with gun, then get talked out of it and you only find out when the whole thing is over that there was a gun – no apprehension of imminent harm

(ii) Mom sees someone put gun to kid’s head –no assault claim because the threat isn’t of offensive or harmful contact to her.

(2) Imminent harm – must be NOW(a) D’s act must cause the victim to expect that he is about to be touched.(b) The imminence question is for the jury.(c) The threat of future contact does not satisfy the imminence requirement.

(i) For a threat to be an assault, has to be such that a reasonable hearer of the threat would believe that the threatened contact is imminent

(ii) A verbal threat constitutes assault when the circumstances of the threat make the victim reasonably apprehend an imminent harmful or offensive touching.

c) Allen v. Hannaford : Landlord was upset because Tenant owed money so pulls a gun on T and movers to prevent the move. Pistol was not loaded, but T did not know that.i) Holding: The pointing of a gun at a person who is ignorant whether or not the gun is loaded constitutes assault

because it is reasonable for a person held at gunpoint to entertain a fear of personal injury.(1) Question isn’t whether has the person caused actual danger, but whether the person thought there was

danger.ii) Hypothetical: If D had threatened that she was going to get a gun and shoot, no claim for assault because no

imminent harm.(a) Not assault if you threaten to do something at some time in the future. Have to have act that caused

imminent harm.d) Battery, but not assault.

i) If someone shoots you from behind – no assault because no apprehension.e) Assault, but not battery

i) Uses a fake gun that looks real – P believes going to get hurt, but D makes no contact.f) Why can you recover relatively easily for assault, but nearly impossible for IIED?

i) Assault: greater propensity for violence, which we want to deter.ii) Because we want to keep option open to deter assaults that can turn into violence really easily, we want to keep

assault law separate from IIED law.

Amanda Lee, Hershovitz – Torts, Fall 2009 43

Page 44: Torts outline

INTENTIONAL INFLICTION OF EMOTIONAL DISTRESS1) IIED imposes liability on actor, who by means of outrageous conduct, intentionally or recklessly causes severe emotional

distress to anothera) Conduct must be intentional: purposeful, wanton, willfulb) Harm that result must also be intentional: must have been intended or recklessly caused

2) 4 elements must coincide to impose liability:a) Conduct intentional or reckless disregard of probability of causing emotional distress

i) Outrageous conduct must be undertaken specifically to cause mental distress, or under the knowledge that such behavior would most certainly cause such mental distress.

b) Conduct must be extreme and outrageousi) To determine, look at:

(1) Conduct(2) Period of time over which conduct occurred(3) Relation b/t P and D(4) D’s knowledge that P susceptible to emotional distress

c) Must be causal connection b/t wrongful conduct and emotional distressd) Emotional distress in question must be severe – so severe that no reasonable person should be expected to endure it

i) Liability only found where conduct so outrageous in character, so extreme in degree, as to go beyond all possible bounds of decency, and be regarded as atrocious and utterly intolerable. (1) If community member heard of act, would say “outrageous!”(2) Societal norms matter – what’s outrageous now may be different than what was outrageous in 1920 – have

to show that it’s outrageous in the context of current societal norms.(3) Not available for mere insults, indignities, threats, or annoyances.

ii) If have emotional distress and as a result have physically injury, both are compensable. IIED is just for emotion component.

3) IIED often pleaded, rarely succeedsa) Ps are in bind, if get life together after an incident, hard to claim IIED b) If P’s life get out of control, court may say reasonable person would endure.

4) Cases:i) Russell v. Salve Regina: P expelled from college b/c of her unwillingness to lose weight

(1) Ct held that P had a claim for IIED.(a) D’s conduct was purposeful.(b) Student-university relationship placed w/in category of relationships which would give rise to a duty to

avoid IIED(c) P’s sensitivity to weight was a known issue

ii) Jones v. Clinton (1) Jones’ claim falls short of IIED standards.

(a) Incident was brief and isolated – didn’t result in any physical harm or severe distress. D’s conduct abandoned when P made clear advances were not welcome.

(b) P’s actions don’t portray someone experiencing emotional distress so severe that no reasonable person could be expected to endure it(i) She cont’d going to work, didn’t go see psychiatrist until right before trial, didn’t file compliant, etc.

DEFENSES TO INTENTIONAL TORTS1) STANDARD DEFENSES TO BATTERY AND ASSAULT

a) Affirmative defense: facts that allow D to avoid liability even if the prima facie elements of the tort are showni) D can say was privileged to act as she did

(1) Justification: to claim justification for one’s conduct is to claim that one was entitled to engage in the conduct, notwithstanding its apparent wrongfulness

(2) Excuse: to claim excuse, is to allege that something about D’s condition or circumstances entitles her to an exemption from rules of right conduct(a) Tort law generally reluctant to recognize excuses

ii) Burden usually on tortfeasor(1) Consent – many state require P to prove absence of consent

b) For assault and battery, rule of comparative fault is not available.c) CONSENT

i) Idea that P can’t prevail on her tort claim b/c she has agreed to endure bodily contact, or an apprehension of contract, or a confinement, that would otherwise be tortuous.

ii) Can be communicated expressly (through written or oral statement) or implicitly (through conduct)(1) Implicit consent

Amanda Lee, Hershovitz – Torts, Fall 2009 44

Page 45: Torts outline

(a) Voluntary participation in contact sports(b) History of dealings b/t parties will support finding of implicit consent

(i) A relationship in which horseplay is common(c) Entering crowded trains and buses

(2) Implicit consent defense requires judgments on circumstances associated with P and Ds interactioniii) Consent not defense if:

(1) Fraud and Coercion: Tortfeasor can’t benefit from consent defense if he secures the victim’s consent by misrepresentation or other forms of deceit; same goes for coercion

(2) Victim lacks ability or judgment necessary to give meaningful consent, and (3) A reasonable person in position of tortfeasor would perceive this lack of capacity

(a) This includes lack of meaningful consent due to age – person consenting must be legally able to consent.(b) Some jurisdictions allow minors to consent to some things.

iv) Apparent consent will sometimes be deemed ineffective if D had reason to know that consent will not be freely given(1) Reavis v. Slominski : receptionist that sued dentist for having sex at office party.

(a) Jury could find that her consent wasn’t voluntary b/c she reasonably feared she would lose her job if she didn’t acquiesce.

v) Ct will bar imposition of liability on D only if, on basis of P’s conduct, D actually and reasonably believed that P has consented to the contact

vi) If there’s been actual consent but D didn’t know about it, D still gets consent defense.vii) Koffman v. Garnett : Coach was mad at his team’s poor tackling, so he demonstrated tackling on a player – told

player to stand still and motionless, picked him up, and threw him to the ground. Player was 13.(1) P implicitly consented to battery by voluntarily participating in football – consented to contact with players

of like age and experience. Didn’t consent to tackling by adult coaches. In past, D had never tackled players before.

(2) “Battery by the ground”: if D causes P to impact the ground(3) Holding: Joining the football team was only consenting to being tackled by other young boys, not the coach.

Not a defense to battery – this contact was outside the scope of consent.(a) Not an assault b/c boy had no apprehension that the hit was coming.

(4) D can’t argue that he was privilege to commit battery on ground that touching was in P’s best interests and therefore P would’ve or should’ve consented.

2) SELF-DEFENSE AND DEFENSE OF OTHERSa) Self-defense available to victim who actually and reasonably believes it’s necessary to injure another to avoid

imminent injuries to herselfi) Reasonableness based on circumstancesii) Reasonable perception of harm and reasonableness in appropriate/proportional response to perceived threat

b) Applies when injury threatened consists of physical harm, inappropriate touching or confinement.i) Doesn’t apply to defamation or distress, or nonthreatening provocations like taunting or teasing

c) Many jurisdictions, person who starts confrontation can’t use self-defensed) Deadly force: if one actually believes that one can safely retreat from a confrontation that would otherwise justify

the use of deadly force, one isn’t justified in using deadly force.i) Other states have adopted “stand your ground” laws – expands privilege of deadly force

(1) FL: With certain exceptions, grants D nonrebuttable presumption that she acted on basis of reasonable apprehension of imminent death or seriously bodily harm if her victim:(a) Was unlawfully and forcibly entering residence, occupied vehicle(b) And D knew or had reason to believe that an unlawful and forcible entry was occurring.

e) Ex: Haeussler v. DeLorretto: P went to D’s house to se if his dog had gone there. When D opened the door, P’s dog rushed out. P got mad and started yelling, advanced threateningly. D struck him once.i) Holding: one involved in an altercation has the right to use force necessary to protect himself from bodily injury.

(1) Question of amount of force justifiable under circumstances is to be determined by trier of fact.(2) Here, enough evidence that D acted reasonably in perceiving imminent threat and used appropriate level of

force in responding to it.f) Defense of Others: operates basically under the same rules that govern the privilege of self-defense

i) As long as D actually and reasonably believes that injuring P is necessary to avoid imminent injury of the requisite type (imminent, and of a sort proportional to force used) to one or more third parties, D is privileged to use proportionate force against P to prevent such injuries

Amanda Lee, Hershovitz – Torts, Fall 2009 45